SAT Exam  >  SAT Tests  >  Digital SAT Mock Test Series 2024  >  OneTime: Digital SAT Mock Test - 6 - SAT MCQ

OneTime: Digital SAT Mock Test - 6 - SAT MCQ


Test Description

30 Questions MCQ Test Digital SAT Mock Test Series 2024 - OneTime: Digital SAT Mock Test - 6

OneTime: Digital SAT Mock Test - 6 for SAT 2024 is part of Digital SAT Mock Test Series 2024 preparation. The OneTime: Digital SAT Mock Test - 6 questions and answers have been prepared according to the SAT exam syllabus.The OneTime: Digital SAT Mock Test - 6 MCQs are made for SAT 2024 Exam. Find important definitions, questions, notes, meanings, examples, exercises, MCQs and online tests for OneTime: Digital SAT Mock Test - 6 below.
Solutions of OneTime: Digital SAT Mock Test - 6 questions in English are available as part of our Digital SAT Mock Test Series 2024 for SAT & OneTime: Digital SAT Mock Test - 6 solutions in Hindi for Digital SAT Mock Test Series 2024 course. Download more important topics, notes, lectures and mock test series for SAT Exam by signing up for free. Attempt OneTime: Digital SAT Mock Test - 6 | 154 questions in 180 minutes | Mock test for SAT preparation | Free important questions MCQ to study Digital SAT Mock Test Series 2024 for SAT Exam | Download free PDF with solutions
OneTime: Digital SAT Mock Test - 6 - Question 1

Question based on the following passage and supplementary material.
Passage 1 is adapted from Nicholas Heidorn, “The Enduring Political Illusion of Farm Subsidies.” ©2004 The Independent Institute. Originally Published August 18, 2004 in the San Francisco Chronicle. Passage 2 is ©2015 by Mark Anestis. Since 1922, the U.S. government has subsidized the agricultural industry by supporting the price of crops (commodity subsidies), paying farmers let their fields go fallow (conservation subsidies), helping farmers purchase crop insurance (crop insurance subsidies), and compensating farmers for uninsured losses due to disasters (disaster subsidies). The following passages discuss these programs.

Passage 1
Something is rotten down on the farm. A recent
General Accounting Office study found that the
U.S. farm subsidy program, a multibillion-dollar
system of direct payments to American farmers,
(5) uses administrators who are ill-trained and
poorly monitored, and who give away millions
of taxpayer dollars to farmers who are actually
ineligible for the program. This report should
horrify lawmakers, but it probably won’t.
(10) From 1995 to 2002, the United States Congress
doled out more than $114 billion to farmers. Why?
One misconception is that subsidies are
a boon to consumers because they lower food
prices. This ignores the fact that consumers are 
(15) also paying for these subsidies through taxes.
Because of inefficiencies in the program, we
taxpayers will pay more in taxes than we will ever
get back in lower corn or wheat prices.
In fact, farm subsidies are not even intended
(20) to reduce food prices significantly. When prices
are too low, farmers lose money. To prevent this
situation, Congress also pays farmers additional
“conservation subsidies” to leave their land fallow,
thereby lowering supply and boosting prices again.
(25) We’re taxed to lower prices, and then taxed to raise them again.
Another myth is that subsidies increase
exports, and thereby benefit the American
economy, by lowering the price of farm products
(30) and so making them more attractive to foreign
consumers. This ignores two realities. First,
farm subsidies transfer wealth from taxpayers
to foreign consumers just as efficiently as
they transfer wealth to domestic consumers.
(35) Second, farm subsidies are actually harming
American exporters. In March 2005, the World
Trade Organization ruled that American cotton
subsidies violated global free-trade rules, which
could lead to billions of dollars in retaliatory
(40) tariffs or penalties.
The worst misconception is that we need these
subsidies to save the small family farmer. Indeed,
according to a 2009 poll, about 77 percent of
Americans support giving subsidies to small family
(45) farms. But according to the Environmental Working
Group, 71 percent of farm subsidies go to the top
10 percent of beneficiaries, almost all of which are
large corporate farms. By subsidizing these rich
farmers, we actually make it much harder for the
(50) small family farmers to compete, not to mention
the millions of impoverished third world farmers
who rely on farming for their livelihood.
Rich corporate farmers are an enormously
powerful lobby in American politics. Agribusines
(55) and farm insurance lobbies pump nearly $100
million into political campaigns every year, and
the floodgates show no sign of closing. So don’t be
surprised if the GAO’s reports of mismanagement
and waste go unheeded. Politicians like their
(60) payouts almost as much as the big farmers and
their insurance companies do.

Passage 2
The critics of the U.S. farm subsidy program fail
to recognize just how vital these subsidies really
are. They are not as burdensome to American
(65) taxpayers as the critics claim, and indeed provide
important benefits. By protecting farmers from
damaging fluctuations in commodity prices due
to weather disasters or market disruptions, these
subsidies help sustain a vital American industry.
(70) At the same time, they protect consumers from
price spikes that can accompany steep drops in
crop inventories. Before price supports became
common in the 20th century, crop failures 
devastated the lives of farmers and consumers with
(75) horrifying frequency.
Opponents say that subsidies distort the
free market and create surpluses in supply. But
halting subsidies would allow regular shortfalls,
which are far more damaging. The year-to-year
(80) carryover of these surpluses protects farmers
from low prices and consumers from high prices.
Another misconception is that subsidies
only benefit the producers. In fact, they help
many related industries as well, including food
(85) processing, distribution, and marketing, chiefly
by helping to lower the cost of production. And,
of course, the consumers receive the benefit of
lower prices.
When assessing the costs and benefits of
(90) farm payments, it is important to compare these
subsidies to those of other industrialized nations.
American farmers receive an average of just 20% of
their incomes from subsidies, compared to 70% for
farmers from some other countries. The European
(95) Union spends about five times what the United
States spends on farm subsidies, amounting to
45% of the EU budget, compared to less than 1%
of the U.S. federal budget. Although the U.S. farm
subsidies programs are not perfect, they provide
(100) enormous benefits not only to farms but also
to associated industries employing millions of
people and to nearly every American consumer.

Q. Both passages acknowledge the effectiveness of U.S. farm subsidies in

Detailed Solution for OneTime: Digital SAT Mock Test - 6 - Question 1

In lines 10–25, the author of Passage 1 discusses how farm subsidies are used both to lower food prices (by subsidizing farmers) when prices get too high, and to raise them (by paying farmers to leave their land fallow) when prices get too low. Although he disputes that these efforts to stabilize prices are worth the cost, he does indicate that they work. In the first paragraph of Passage 2 (lines 60–73), the author indicates that farm subsidies protect consumers from price spikes (lines 69–70).

OneTime: Digital SAT Mock Test - 6 - Question 2

Question based on the following passage and supplementary material.
Passage 1 is adapted from Nicholas Heidorn, “The Enduring Political Illusion of Farm Subsidies.” ©2004 The Independent Institute. Originally Published August 18, 2004 in the San Francisco Chronicle. Passage 2 is ©2015 by Mark Anestis. Since 1922, the U.S. government has subsidized the agricultural industry by supporting the price of crops (commodity subsidies), paying farmers let their fields go fallow (conservation subsidies), helping farmers purchase crop insurance (crop insurance subsidies), and compensating farmers for uninsured losses due to disasters (disaster subsidies). The following passages discuss these programs.

Passage 1
Something is rotten down on the farm. A recent
General Accounting Office study found that the
U.S. farm subsidy program, a multibillion-dollar
system of direct payments to American farmers,
(5) uses administrators who are ill-trained and
poorly monitored, and who give away millions
of taxpayer dollars to farmers who are actually
ineligible for the program. This report should
horrify lawmakers, but it probably won’t.
(10) From 1995 to 2002, the United States Congress
doled out more than $114 billion to farmers. Why?
One misconception is that subsidies are
a boon to consumers because they lower food
prices. This ignores the fact that consumers are 
(15) also paying for these subsidies through taxes.
Because of inefficiencies in the program, we
taxpayers will pay more in taxes than we will ever
get back in lower corn or wheat prices.
In fact, farm subsidies are not even intended
(20) to reduce food prices significantly. When prices
are too low, farmers lose money. To prevent this
situation, Congress also pays farmers additional
“conservation subsidies” to leave their land fallow,
thereby lowering supply and boosting prices again.
(25) We’re taxed to lower prices, and then taxed to raise them again.
Another myth is that subsidies increase
exports, and thereby benefit the American
economy, by lowering the price of farm products
(30) and so making them more attractive to foreign
consumers. This ignores two realities. First,
farm subsidies transfer wealth from taxpayers
to foreign consumers just as efficiently as
they transfer wealth to domestic consumers.
(35) Second, farm subsidies are actually harming
American exporters. In March 2005, the World
Trade Organization ruled that American cotton
subsidies violated global free-trade rules, which
could lead to billions of dollars in retaliatory
(40) tariffs or penalties.
The worst misconception is that we need these
subsidies to save the small family farmer. Indeed,
according to a 2009 poll, about 77 percent of
Americans support giving subsidies to small family
(45) farms. But according to the Environmental Working
Group, 71 percent of farm subsidies go to the top
10 percent of beneficiaries, almost all of which are
large corporate farms. By subsidizing these rich
farmers, we actually make it much harder for the
(50) small family farmers to compete, not to mention
the millions of impoverished third world farmers
who rely on farming for their livelihood.
Rich corporate farmers are an enormously
powerful lobby in American politics. Agribusines
(55) and farm insurance lobbies pump nearly $100
million into political campaigns every year, and
the floodgates show no sign of closing. So don’t be
surprised if the GAO’s reports of mismanagement
and waste go unheeded. Politicians like their
(60) payouts almost as much as the big farmers and
their insurance companies do.

Passage 2
The critics of the U.S. farm subsidy program fail
to recognize just how vital these subsidies really
are. They are not as burdensome to American
(65) taxpayers as the critics claim, and indeed provide
important benefits. By protecting farmers from
damaging fluctuations in commodity prices due
to weather disasters or market disruptions, these
subsidies help sustain a vital American industry.
(70) At the same time, they protect consumers from
price spikes that can accompany steep drops in
crop inventories. Before price supports became
common in the 20th century, crop failures 
devastated the lives of farmers and consumers with
(75) horrifying frequency.
Opponents say that subsidies distort the
free market and create surpluses in supply. But
halting subsidies would allow regular shortfalls,
which are far more damaging. The year-to-year
(80) carryover of these surpluses protects farmers
from low prices and consumers from high prices.
Another misconception is that subsidies
only benefit the producers. In fact, they help
many related industries as well, including food
(85) processing, distribution, and marketing, chiefly
by helping to lower the cost of production. And,
of course, the consumers receive the benefit of
lower prices.
When assessing the costs and benefits of
(90) farm payments, it is important to compare these
subsidies to those of other industrialized nations.
American farmers receive an average of just 20% of
their incomes from subsidies, compared to 70% for
farmers from some other countries. The European
(95) Union spends about five times what the United
States spends on farm subsidies, amounting to
45% of the EU budget, compared to less than 1%
of the U.S. federal budget. Although the U.S. farm
subsidies programs are not perfect, they provide
(100) enormous benefits not only to farms but also
to associated industries employing millions of
people and to nearly every American consumer.

Q. The first sentence of Passage 1 refers primarily to the author’s belief that

Detailed Solution for OneTime: Digital SAT Mock Test - 6 - Question 2

In the first paragraph of Passage 1, the statement that something is rotten down on the farm (line 1) introduces the author’s discussion of the U.S. farm subsidies program, which he claims gives away millions of taxpayer dollars to farmers who are actually ineligible for the program (lines 6–8) and is rife with inefficiencies (line 16).

1 Crore+ students have signed up on EduRev. Have you? Download the App
OneTime: Digital SAT Mock Test - 6 - Question 3

Question based on the following passage and supplementary material.
Passage 1 is adapted from Nicholas Heidorn, “The Enduring Political Illusion of Farm Subsidies.” ©2004 The Independent Institute. Originally Published August 18, 2004 in the San Francisco Chronicle. Passage 2 is ©2015 by Mark Anestis. Since 1922, the U.S. government has subsidized the agricultural industry by supporting the price of crops (commodity subsidies), paying farmers let their fields go fallow (conservation subsidies), helping farmers purchase crop insurance (crop insurance subsidies), and compensating farmers for uninsured losses due to disasters (disaster subsidies). The following passages discuss these programs.

Passage 1
Something is rotten down on the farm. A recent
General Accounting Office study found that the
U.S. farm subsidy program, a multibillion-dollar
system of direct payments to American farmers,
(5) uses administrators who are ill-trained and
poorly monitored, and who give away millions
of taxpayer dollars to farmers who are actually
ineligible for the program. This report should
horrify lawmakers, but it probably won’t.
(10) From 1995 to 2002, the United States Congress
doled out more than $114 billion to farmers. Why?
One misconception is that subsidies are
a boon to consumers because they lower food
prices. This ignores the fact that consumers are 
(15) also paying for these subsidies through taxes.
Because of inefficiencies in the program, we
taxpayers will pay more in taxes than we will ever
get back in lower corn or wheat prices.
In fact, farm subsidies are not even intended
(20) to reduce food prices significantly. When prices
are too low, farmers lose money. To prevent this
situation, Congress also pays farmers additional
“conservation subsidies” to leave their land fallow,
thereby lowering supply and boosting prices again.
(25) We’re taxed to lower prices, and then taxed to raise them again.
Another myth is that subsidies increase
exports, and thereby benefit the American
economy, by lowering the price of farm products
(30) and so making them more attractive to foreign
consumers. This ignores two realities. First,
farm subsidies transfer wealth from taxpayers
to foreign consumers just as efficiently as
they transfer wealth to domestic consumers.
(35) Second, farm subsidies are actually harming
American exporters. In March 2005, the World
Trade Organization ruled that American cotton
subsidies violated global free-trade rules, which
could lead to billions of dollars in retaliatory
(40) tariffs or penalties.
The worst misconception is that we need these
subsidies to save the small family farmer. Indeed,
according to a 2009 poll, about 77 percent of
Americans support giving subsidies to small family
(45) farms. But according to the Environmental Working
Group, 71 percent of farm subsidies go to the top
10 percent of beneficiaries, almost all of which are
large corporate farms. By subsidizing these rich
farmers, we actually make it much harder for the
(50) small family farmers to compete, not to mention
the millions of impoverished third world farmers
who rely on farming for their livelihood.
Rich corporate farmers are an enormously
powerful lobby in American politics. Agribusines
(55) and farm insurance lobbies pump nearly $100
million into political campaigns every year, and
the floodgates show no sign of closing. So don’t be
surprised if the GAO’s reports of mismanagement
and waste go unheeded. Politicians like their
(60) payouts almost as much as the big farmers and
their insurance companies do.

Passage 2
The critics of the U.S. farm subsidy program fail
to recognize just how vital these subsidies really
are. They are not as burdensome to American
(65) taxpayers as the critics claim, and indeed provide
important benefits. By protecting farmers from
damaging fluctuations in commodity prices due
to weather disasters or market disruptions, these
subsidies help sustain a vital American industry.
(70) At the same time, they protect consumers from
price spikes that can accompany steep drops in
crop inventories. Before price supports became
common in the 20th century, crop failures 
devastated the lives of farmers and consumers with
(75) horrifying frequency.
Opponents say that subsidies distort the
free market and create surpluses in supply. But
halting subsidies would allow regular shortfalls,
which are far more damaging. The year-to-year
(80) carryover of these surpluses protects farmers
from low prices and consumers from high prices.
Another misconception is that subsidies
only benefit the producers. In fact, they help
many related industries as well, including food
(85) processing, distribution, and marketing, chiefly
by helping to lower the cost of production. And,
of course, the consumers receive the benefit of
lower prices.
When assessing the costs and benefits of
(90) farm payments, it is important to compare these
subsidies to those of other industrialized nations.
American farmers receive an average of just 20% of
their incomes from subsidies, compared to 70% for
farmers from some other countries. The European
(95) Union spends about five times what the United
States spends on farm subsidies, amounting to
45% of the EU budget, compared to less than 1%
of the U.S. federal budget. Although the U.S. farm
subsidies programs are not perfect, they provide
(100) enormous benefits not only to farms but also
to associated industries employing millions of
people and to nearly every American consumer.

Q. The author of Passage 2 would most likely regard the “taxes” mentioned in line 15 as

Detailed Solution for OneTime: Digital SAT Mock Test - 6 - Question 3

The thesis of Passage 2 is that U.S. farm subsidies are vital (line 63) to both farmers and American consumers. Therefore, he regards the taxes (line 15) we pay for these subsidies to be a worthwhile expenditure.

OneTime: Digital SAT Mock Test - 6 - Question 4

Question are based on the following passage and supplementary material.
Passage 1 is adapted from Nicholas Heidorn, “The Enduring Political Illusion of Farm Subsidies.” ©2004 The Independent Institute. Originally Published August 18, 2004 in the San Francisco Chronicle. Passage 2 is ©2015 by Mark Anestis. Since 1922, the U.S. government has subsidized the agricultural industry by supporting the price of crops (commodity subsidies), paying farmers let their fields go fallow (conservation subsidies), helping farmers purchase crop insurance (crop insurance subsidies), and compensating farmers for uninsured losses due to disasters (disaster subsidies). The following passages discuss these programs.

Passage 1
Something is rotten down on the farm. A recent
General Accounting Office study found that the
U.S. farm subsidy program, a multibillion-dollar
system of direct payments to American farmers,
(5) uses administrators who are ill-trained and
poorly monitored, and who give away millions
of taxpayer dollars to farmers who are actually
ineligible for the program. This report should
horrify lawmakers, but it probably won’t.
(10) From 1995 to 2002, the United States Congress
doled out more than $114 billion to farmers. Why?
One misconception is that subsidies are
a boon to consumers because they lower food
prices. This ignores the fact that consumers are 
(15) also paying for these subsidies through taxes.
Because of inefficiencies in the program, we
taxpayers will pay more in taxes than we will ever
get back in lower corn or wheat prices.
In fact, farm subsidies are not even intended
(20) to reduce food prices significantly. When prices
are too low, farmers lose money. To prevent this
situation, Congress also pays farmers additional
“conservation subsidies” to leave their land fallow,
thereby lowering supply and boosting prices again.
(25) We’re taxed to lower prices, and then taxed to raise them again.
Another myth is that subsidies increase
exports, and thereby benefit the American
economy, by lowering the price of farm products
(30) and so making them more attractive to foreign
consumers. This ignores two realities. First,
farm subsidies transfer wealth from taxpayers
to foreign consumers just as efficiently as
they transfer wealth to domestic consumers.
(35) Second, farm subsidies are actually harming
American exporters. In March 2005, the World
Trade Organization ruled that American cotton
subsidies violated global free-trade rules, which
could lead to billions of dollars in retaliatory
(40) tariffs or penalties.
The worst misconception is that we need these
subsidies to save the small family farmer. Indeed,
according to a 2009 poll, about 77 percent of
Americans support giving subsidies to small family
(45) farms. But according to the Environmental Working
Group, 71 percent of farm subsidies go to the top
10 percent of beneficiaries, almost all of which are
large corporate farms. By subsidizing these rich
farmers, we actually make it much harder for the
(50) small family farmers to compete, not to mention
the millions of impoverished third world farmers
who rely on farming for their livelihood.
Rich corporate farmers are an enormously
powerful lobby in American politics. Agribusines
(55) and farm insurance lobbies pump nearly $100
million into political campaigns every year, and
the floodgates show no sign of closing. So don’t be
surprised if the GAO’s reports of mismanagement
and waste go unheeded. Politicians like their
(60) payouts almost as much as the big farmers and
their insurance companies do.

Passage 2
The critics of the U.S. farm subsidy program fail
to recognize just how vital these subsidies really
are. They are not as burdensome to American
(65) taxpayers as the critics claim, and indeed provide
important benefits. By protecting farmers from
damaging fluctuations in commodity prices due
to weather disasters or market disruptions, these
subsidies help sustain a vital American industry.
(70) At the same time, they protect consumers from
price spikes that can accompany steep drops in
crop inventories. Before price supports became
common in the 20th century, crop failures 
devastated the lives of farmers and consumers with
(75) horrifying frequency.
Opponents say that subsidies distort the
free market and create surpluses in supply. But
halting subsidies would allow regular shortfalls,
which are far more damaging. The year-to-year
(80) carryover of these surpluses protects farmers
from low prices and consumers from high prices.
Another misconception is that subsidies
only benefit the producers. In fact, they help
many related industries as well, including food
(85) processing, distribution, and marketing, chiefly
by helping to lower the cost of production. And,
of course, the consumers receive the benefit of
lower prices.
When assessing the costs and benefits of
(90) farm payments, it is important to compare these
subsidies to those of other industrialized nations.
American farmers receive an average of just 20% of
their incomes from subsidies, compared to 70% for
farmers from some other countries. The European
(95) Union spends about five times what the United
States spends on farm subsidies, amounting to
45% of the EU budget, compared to less than 1%
of the U.S. federal budget. Although the U.S. farm
subsidies programs are not perfect, they provide
(100) enormous benefits not only to farms but also
to associated industries employing millions of
people and to nearly every American consumer.

Q. The author of Passage 1 believes that the GAO report “probably won’t” (line 9) horrify lawmakers because

Detailed Solution for OneTime: Digital SAT Mock Test - 6 - Question 4

In lines 8–9, the author of Passage 1 states that the report about corruption and incompetence in the U.S. farm subsidies program should horrify lawmakers, but it probably won’t. He explains why in the last paragraph (lines 53–56): Rich corporate farmers are an enormously powerful lobby in American politics, contributing nearly $100 million into political campaigns every year.

OneTime: Digital SAT Mock Test - 6 - Question 5

Question are based on the following passage and supplementary material.
Passage 1 is adapted from Nicholas Heidorn, “The Enduring Political Illusion of Farm Subsidies.” ©2004 The Independent Institute. Originally Published August 18, 2004 in the San Francisco Chronicle. Passage 2 is ©2015 by Mark Anestis. Since 1922, the U.S. government has subsidized the agricultural industry by supporting the price of crops (commodity subsidies), paying farmers let their fields go fallow (conservation subsidies), helping farmers purchase crop insurance (crop insurance subsidies), and compensating farmers for uninsured losses due to disasters (disaster subsidies). The following passages discuss these programs.

Passage 1
Something is rotten down on the farm. A recent
General Accounting Office study found that the
U.S. farm subsidy program, a multibillion-dollar
system of direct payments to American farmers,
(5) uses administrators who are ill-trained and
poorly monitored, and who give away millions
of taxpayer dollars to farmers who are actually
ineligible for the program. This report should
horrify lawmakers, but it probably won’t.
(10) From 1995 to 2002, the United States Congress
doled out more than $114 billion to farmers. Why?
One misconception is that subsidies are
a boon to consumers because they lower food
prices. This ignores the fact that consumers are 
(15) also paying for these subsidies through taxes.
Because of inefficiencies in the program, we
taxpayers will pay more in taxes than we will ever
get back in lower corn or wheat prices.
In fact, farm subsidies are not even intended
(20) to reduce food prices significantly. When prices
are too low, farmers lose money. To prevent this
situation, Congress also pays farmers additional
“conservation subsidies” to leave their land fallow,
thereby lowering supply and boosting prices again.
(25) We’re taxed to lower prices, and then taxed to raise them again.
Another myth is that subsidies increase
exports, and thereby benefit the American
economy, by lowering the price of farm products
(30) and so making them more attractive to foreign
consumers. This ignores two realities. First,
farm subsidies transfer wealth from taxpayers
to foreign consumers just as efficiently as
they transfer wealth to domestic consumers.
(35) Second, farm subsidies are actually harming
American exporters. In March 2005, the World
Trade Organization ruled that American cotton
subsidies violated global free-trade rules, which
could lead to billions of dollars in retaliatory
(40) tariffs or penalties.
The worst misconception is that we need these
subsidies to save the small family farmer. Indeed,
according to a 2009 poll, about 77 percent of
Americans support giving subsidies to small family
(45) farms. But according to the Environmental Working
Group, 71 percent of farm subsidies go to the top
10 percent of beneficiaries, almost all of which are
large corporate farms. By subsidizing these rich
farmers, we actually make it much harder for the
(50) small family farmers to compete, not to mention
the millions of impoverished third world farmers
who rely on farming for their livelihood.
Rich corporate farmers are an enormously
powerful lobby in American politics. Agribusines
(55) and farm insurance lobbies pump nearly $100
million into political campaigns every year, and
the floodgates show no sign of closing. So don’t be
surprised if the GAO’s reports of mismanagement
and waste go unheeded. Politicians like their
(60) payouts almost as much as the big farmers and
their insurance companies do.

Passage 2
The critics of the U.S. farm subsidy program fail
to recognize just how vital these subsidies really
are. They are not as burdensome to American
(65) taxpayers as the critics claim, and indeed provide
important benefits. By protecting farmers from
damaging fluctuations in commodity prices due
to weather disasters or market disruptions, these
subsidies help sustain a vital American industry.
(70) At the same time, they protect consumers from
price spikes that can accompany steep drops in
crop inventories. Before price supports became
common in the 20th century, crop failures 
devastated the lives of farmers and consumers with
(75) horrifying frequency.
Opponents say that subsidies distort the
free market and create surpluses in supply. But
halting subsidies would allow regular shortfalls,
which are far more damaging. The year-to-year
(80) carryover of these surpluses protects farmers
from low prices and consumers from high prices.
Another misconception is that subsidies
only benefit the producers. In fact, they help
many related industries as well, including food
(85) processing, distribution, and marketing, chiefly
by helping to lower the cost of production. And,
of course, the consumers receive the benefit of
lower prices.
When assessing the costs and benefits of
(90) farm payments, it is important to compare these
subsidies to those of other industrialized nations.
American farmers receive an average of just 20% of
their incomes from subsidies, compared to 70% for
farmers from some other countries. The European
(95) Union spends about five times what the United
States spends on farm subsidies, amounting to
45% of the EU budget, compared to less than 1%
of the U.S. federal budget. Although the U.S. farm
subsidies programs are not perfect, they provide
(100) enormous benefits not only to farms but also
to associated industries employing millions of
people and to nearly every American consumer.

Q. Which of the following provides the strongest evidence for the answer to the previous question?

Detailed Solution for OneTime: Digital SAT Mock Test - 6 - Question 5

As the explanation to question 4 explains, the evidence for this answer is found in the last paragraph, particularly lines 54–57.

OneTime: Digital SAT Mock Test - 6 - Question 6

Question are based on the following passage and supplementary material.
Passage 1 is adapted from Nicholas Heidorn, “The Enduring Political Illusion of Farm Subsidies.” ©2004 The Independent Institute. Originally Published August 18, 2004 in the San Francisco Chronicle. Passage 2 is ©2015 by Mark Anestis. Since 1922, the U.S. government has subsidized the agricultural industry by supporting the price of crops (commodity subsidies), paying farmers let their fields go fallow (conservation subsidies), helping farmers purchase crop insurance (crop insurance subsidies), and compensating farmers for uninsured losses due to disasters (disaster subsidies). The following passages discuss these programs.

Passage 1
Something is rotten down on the farm. A recent
General Accounting Office study found that the
U.S. farm subsidy program, a multibillion-dollar
system of direct payments to American farmers,
(5) uses administrators who are ill-trained and
poorly monitored, and who give away millions
of taxpayer dollars to farmers who are actually
ineligible for the program. This report should
horrify lawmakers, but it probably won’t.
(10) From 1995 to 2002, the United States Congress
doled out more than $114 billion to farmers. Why?
One misconception is that subsidies are
a boon to consumers because they lower food
prices. This ignores the fact that consumers are 
(15) also paying for these subsidies through taxes.
Because of inefficiencies in the program, we
taxpayers will pay more in taxes than we will ever
get back in lower corn or wheat prices.
In fact, farm subsidies are not even intended
(20) to reduce food prices significantly. When prices
are too low, farmers lose money. To prevent this
situation, Congress also pays farmers additional
“conservation subsidies” to leave their land fallow,
thereby lowering supply and boosting prices again.
(25) We’re taxed to lower prices, and then taxed to raise them again.
Another myth is that subsidies increase
exports, and thereby benefit the American
economy, by lowering the price of farm products
(30) and so making them more attractive to foreign
consumers. This ignores two realities. First,
farm subsidies transfer wealth from taxpayers
to foreign consumers just as efficiently as
they transfer wealth to domestic consumers.
(35) Second, farm subsidies are actually harming
American exporters. In March 2005, the World
Trade Organization ruled that American cotton
subsidies violated global free-trade rules, which
could lead to billions of dollars in retaliatory
(40) tariffs or penalties.
The worst misconception is that we need these
subsidies to save the small family farmer. Indeed,
according to a 2009 poll, about 77 percent of
Americans support giving subsidies to small family
(45) farms. But according to the Environmental Working
Group, 71 percent of farm subsidies go to the top
10 percent of beneficiaries, almost all of which are
large corporate farms. By subsidizing these rich
farmers, we actually make it much harder for the
(50) small family farmers to compete, not to mention
the millions of impoverished third world farmers
who rely on farming for their livelihood.
Rich corporate farmers are an enormously
powerful lobby in American politics. Agribusines
(55) and farm insurance lobbies pump nearly $100
million into political campaigns every year, and
the floodgates show no sign of closing. So don’t be
surprised if the GAO’s reports of mismanagement
and waste go unheeded. Politicians like their
(60) payouts almost as much as the big farmers and
their insurance companies do.

Passage 2
The critics of the U.S. farm subsidy program fail
to recognize just how vital these subsidies really
are. They are not as burdensome to American
(65) taxpayers as the critics claim, and indeed provide
important benefits. By protecting farmers from
damaging fluctuations in commodity prices due
to weather disasters or market disruptions, these
subsidies help sustain a vital American industry.
(70) At the same time, they protect consumers from
price spikes that can accompany steep drops in
crop inventories. Before price supports became
common in the 20th century, crop failures 
devastated the lives of farmers and consumers with
(75) horrifying frequency.
Opponents say that subsidies distort the
free market and create surpluses in supply. But
halting subsidies would allow regular shortfalls,
which are far more damaging. The year-to-year
(80) carryover of these surpluses protects farmers
from low prices and consumers from high prices.
Another misconception is that subsidies
only benefit the producers. In fact, they help
many related industries as well, including food
(85) processing, distribution, and marketing, chiefly
by helping to lower the cost of production. And,
of course, the consumers receive the benefit of
lower prices.
When assessing the costs and benefits of
(90) farm payments, it is important to compare these
subsidies to those of other industrialized nations.
American farmers receive an average of just 20% of
their incomes from subsidies, compared to 70% for
farmers from some other countries. The European
(95) Union spends about five times what the United
States spends on farm subsidies, amounting to
45% of the EU budget, compared to less than 1%
of the U.S. federal budget. Although the U.S. farm
subsidies programs are not perfect, they provide
(100) enormous benefits not only to farms but also
to associated industries employing millions of
people and to nearly every American consumer.

Q. Unlike Passage 1, Passage 2 emphasizes the danger of

Detailed Solution for OneTime: Digital SAT Mock Test - 6 - Question 6

Although the author of Passage 1 does not think that the stabilization of commodity prices is worth the cost of higher taxes (lines 19–25), the author of Passage 2 indicates that price spikes (line 71) can be devastating to both farmers and consumers.

OneTime: Digital SAT Mock Test - 6 - Question 7

Question are based on the following passage and supplementary material.
Passage 1 is adapted from Nicholas Heidorn, “The Enduring Political Illusion of Farm Subsidies.” ©2004 The Independent Institute. Originally Published August 18, 2004 in the San Francisco Chronicle. Passage 2 is ©2015 by Mark Anestis. Since 1922, the U.S. government has subsidized the agricultural industry by supporting the price of crops (commodity subsidies), paying farmers let their fields go fallow (conservation subsidies), helping farmers purchase crop insurance (crop insurance subsidies), and compensating farmers for uninsured losses due to disasters (disaster subsidies). The following passages discuss these programs.

Passage 1
Something is rotten down on the farm. A recent
General Accounting Office study found that the
U.S. farm subsidy program, a multibillion-dollar
system of direct payments to American farmers,
(5) uses administrators who are ill-trained and
poorly monitored, and who give away millions
of taxpayer dollars to farmers who are actually
ineligible for the program. This report should
horrify lawmakers, but it probably won’t.
(10) From 1995 to 2002, the United States Congress
doled out more than $114 billion to farmers. Why?
One misconception is that subsidies are
a boon to consumers because they lower food
prices. This ignores the fact that consumers are 
(15) also paying for these subsidies through taxes.
Because of inefficiencies in the program, we
taxpayers will pay more in taxes than we will ever
get back in lower corn or wheat prices.
In fact, farm subsidies are not even intended
(20) to reduce food prices significantly. When prices
are too low, farmers lose money. To prevent this
situation, Congress also pays farmers additional
“conservation subsidies” to leave their land fallow,
thereby lowering supply and boosting prices again.
(25) We’re taxed to lower prices, and then taxed to raise them again.
Another myth is that subsidies increase
exports, and thereby benefit the American
economy, by lowering the price of farm products
(30) and so making them more attractive to foreign
consumers. This ignores two realities. First,
farm subsidies transfer wealth from taxpayers
to foreign consumers just as efficiently as
they transfer wealth to domestic consumers.
(35) Second, farm subsidies are actually harming
American exporters. In March 2005, the World
Trade Organization ruled that American cotton
subsidies violated global free-trade rules, which
could lead to billions of dollars in retaliatory
(40) tariffs or penalties.
The worst misconception is that we need these
subsidies to save the small family farmer. Indeed,
according to a 2009 poll, about 77 percent of
Americans support giving subsidies to small family
(45) farms. But according to the Environmental Working
Group, 71 percent of farm subsidies go to the top
10 percent of beneficiaries, almost all of which are
large corporate farms. By subsidizing these rich
farmers, we actually make it much harder for the
(50) small family farmers to compete, not to mention
the millions of impoverished third world farmers
who rely on farming for their livelihood.
Rich corporate farmers are an enormously
powerful lobby in American politics. Agribusines
(55) and farm insurance lobbies pump nearly $100
million into political campaigns every year, and
the floodgates show no sign of closing. So don’t be
surprised if the GAO’s reports of mismanagement
and waste go unheeded. Politicians like their
(60) payouts almost as much as the big farmers and
their insurance companies do.

Passage 2
The critics of the U.S. farm subsidy program fail
to recognize just how vital these subsidies really
are. They are not as burdensome to American
(65) taxpayers as the critics claim, and indeed provide
important benefits. By protecting farmers from
damaging fluctuations in commodity prices due
to weather disasters or market disruptions, these
subsidies help sustain a vital American industry.
(70) At the same time, they protect consumers from
price spikes that can accompany steep drops in
crop inventories. Before price supports became
common in the 20th century, crop failures 
devastated the lives of farmers and consumers with
(75) horrifying frequency.
Opponents say that subsidies distort the
free market and create surpluses in supply. But
halting subsidies would allow regular shortfalls,
which are far more damaging. The year-to-year
(80) carryover of these surpluses protects farmers
from low prices and consumers from high prices.
Another misconception is that subsidies
only benefit the producers. In fact, they help
many related industries as well, including food
(85) processing, distribution, and marketing, chiefly
by helping to lower the cost of production. And,
of course, the consumers receive the benefit of
lower prices.
When assessing the costs and benefits of
(90) farm payments, it is important to compare these
subsidies to those of other industrialized nations.
American farmers receive an average of just 20% of
their incomes from subsidies, compared to 70% for
farmers from some other countries. The European
(95) Union spends about five times what the United
States spends on farm subsidies, amounting to
45% of the EU budget, compared to less than 1%
of the U.S. federal budget. Although the U.S. farm
subsidies programs are not perfect, they provide
(100) enormous benefits not only to farms but also
to associated industries employing millions of
people and to nearly every American consumer.

Q. Passage 1 mentions the results of the 2009 poll (lines 42–45) primarily to

Detailed Solution for OneTime: Digital SAT Mock Test - 6 - Question 7

The 2009 poll cited in lines 42–45 indicates that most Americans support farm subsidies for small family farms, confirming the author’s statement that Americans feel that we need these subsidies to save the small family farmer. Choice (B) is incorrect because, although the author himself goes on to refute this misconception, the results of the poll do not. Rather, they confirm a general sentiment. Choice (C) is incorrect because the poll does not indicate any shift away from the discussion about the ineffectiveness of the U.S. farm subsidies program. Choice (D) is incorrect because the word Indeed (line 42) indicates that this result is unsurprising to him.

OneTime: Digital SAT Mock Test - 6 - Question 8

Question are based on the following passage and supplementary material.
Passage 1 is adapted from Nicholas Heidorn, “The Enduring Political Illusion of Farm Subsidies.” ©2004 The Independent Institute. Originally Published August 18, 2004 in the San Francisco Chronicle. Passage 2 is ©2015 by Mark Anestis. Since 1922, the U.S. government has subsidized the agricultural industry by supporting the price of crops (commodity subsidies), paying farmers let their fields go fallow (conservation subsidies), helping farmers purchase crop insurance (crop insurance subsidies), and compensating farmers for uninsured losses due to disasters (disaster subsidies). The following passages discuss these programs.

Passage 1
Something is rotten down on the farm. A recent
General Accounting Office study found that the
U.S. farm subsidy program, a multibillion-dollar
system of direct payments to American farmers,
(5) uses administrators who are ill-trained and
poorly monitored, and who give away millions
of taxpayer dollars to farmers who are actually
ineligible for the program. This report should
horrify lawmakers, but it probably won’t.
(10) From 1995 to 2002, the United States Congress
doled out more than $114 billion to farmers. Why?
One misconception is that subsidies are
a boon to consumers because they lower food
prices. This ignores the fact that consumers are 
(15) also paying for these subsidies through taxes.
Because of inefficiencies in the program, we
taxpayers will pay more in taxes than we will ever
get back in lower corn or wheat prices.
In fact, farm subsidies are not even intended
(20) to reduce food prices significantly. When prices
are too low, farmers lose money. To prevent this
situation, Congress also pays farmers additional
“conservation subsidies” to leave their land fallow,
thereby lowering supply and boosting prices again.
(25) We’re taxed to lower prices, and then taxed to raise them again.
Another myth is that subsidies increase
exports, and thereby benefit the American
economy, by lowering the price of farm products
(30) and so making them more attractive to foreign
consumers. This ignores two realities. First,
farm subsidies transfer wealth from taxpayers
to foreign consumers just as efficiently as
they transfer wealth to domestic consumers.
(35) Second, farm subsidies are actually harming
American exporters. In March 2005, the World
Trade Organization ruled that American cotton
subsidies violated global free-trade rules, which
could lead to billions of dollars in retaliatory
(40) tariffs or penalties.
The worst misconception is that we need these
subsidies to save the small family farmer. Indeed,
according to a 2009 poll, about 77 percent of
Americans support giving subsidies to small family
(45) farms. But according to the Environmental Working
Group, 71 percent of farm subsidies go to the top
10 percent of beneficiaries, almost all of which are
large corporate farms. By subsidizing these rich
farmers, we actually make it much harder for the
(50) small family farmers to compete, not to mention
the millions of impoverished third world farmers
who rely on farming for their livelihood.
Rich corporate farmers are an enormously
powerful lobby in American politics. Agribusines
(55) and farm insurance lobbies pump nearly $100
million into political campaigns every year, and
the floodgates show no sign of closing. So don’t be
surprised if the GAO’s reports of mismanagement
and waste go unheeded. Politicians like their
(60) payouts almost as much as the big farmers and
their insurance companies do.

Passage 2
The critics of the U.S. farm subsidy program fail
to recognize just how vital these subsidies really
are. They are not as burdensome to American
(65) taxpayers as the critics claim, and indeed provide
important benefits. By protecting farmers from
damaging fluctuations in commodity prices due
to weather disasters or market disruptions, these
subsidies help sustain a vital American industry.
(70) At the same time, they protect consumers from
price spikes that can accompany steep drops in
crop inventories. Before price supports became
common in the 20th century, crop failures 
devastated the lives of farmers and consumers with
(75) horrifying frequency.
Opponents say that subsidies distort the
free market and create surpluses in supply. But
halting subsidies would allow regular shortfalls,
which are far more damaging. The year-to-year
(80) carryover of these surpluses protects farmers
from low prices and consumers from high prices.
Another misconception is that subsidies
only benefit the producers. In fact, they help
many related industries as well, including food
(85) processing, distribution, and marketing, chiefly
by helping to lower the cost of production. And,
of course, the consumers receive the benefit of
lower prices.
When assessing the costs and benefits of
(90) farm payments, it is important to compare these
subsidies to those of other industrialized nations.
American farmers receive an average of just 20% of
their incomes from subsidies, compared to 70% for
farmers from some other countries. The European
(95) Union spends about five times what the United
States spends on farm subsidies, amounting to
45% of the EU budget, compared to less than 1%
of the U.S. federal budget. Although the U.S. farm
subsidies programs are not perfect, they provide
(100) enormous benefits not only to farms but also
to associated industries employing millions of
people and to nearly every American consumer.

Q. If the author of Passage 1 were to use the data in the graph to support his main thesis, he would most likely mention

Detailed Solution for OneTime: Digital SAT Mock Test - 6 - Question 8

The thesis of Passage 1 is that something is rotten down on the farm (line 1), namely, the fact that, in a recent sevenyear period, the Uited States Congress has doled out more than $114 billion to farmers (lines 10–11) through a program that uses administrators who are ill-trained and poorly monitored (lines 5–6) and that implements programs that are not worthwhile to taxpayers, that are actually harming American exporters (lines 35–36) and that make it much harder for the small family farmers to compete (lines 49–50). The graph in Figure 1, however, shows about a 40% decline in these subsidies from 2000 to 2012, perhaps undercutting the author’s claim that these subsidies are an overall burden on the American taxpayer.
He would most likely, then, choose to focus on the component of these subsidies that has grown significantly in the 15 years indicated on this graph, namely, crop insurance subsidies, which have expanded at a fairly steady rate and grew by about 500% from 1998 to 2012. As the introduction to the passage indicates, this program takes money from taxpayers to help farmers to buy crop insurance, thereby providing direct entitlements not only to farmers but also to insurance companies.
Choice (A), the general decline in total farm subsidies from 2005 to 2012, does not help the author make the point that these subsidies are a burden to American taxpayers. Similarly, choice (B), the overall rate of change in commodity subsidies from 1998 to 2012 does not help his thesis, because after the first several years, the trend is generally downward. Choice (D), the sudden spike in disaster subsidies from 2004 to 2005, also does not support his thesis, because he does not make any particular claims about the benefit of disaster subsidies.

OneTime: Digital SAT Mock Test - 6 - Question 9

Question are based on the following passage and supplementary material.
Passage 1 is adapted from Nicholas Heidorn, “The Enduring Political Illusion of Farm Subsidies.” ©2004 The Independent Institute. Originally Published August 18, 2004 in the San Francisco Chronicle. Passage 2 is ©2015 by Mark Anestis. Since 1922, the U.S. government has subsidized the agricultural industry by supporting the price of crops (commodity subsidies), paying farmers let their fields go fallow (conservation subsidies), helping farmers purchase crop insurance (crop insurance subsidies), and compensating farmers for uninsured losses due to disasters (disaster subsidies). The following passages discuss these programs.

Passage 1
Something is rotten down on the farm. A recent
General Accounting Office study found that the
U.S. farm subsidy program, a multibillion-dollar
system of direct payments to American farmers,
(5) uses administrators who are ill-trained and
poorly monitored, and who give away millions
of taxpayer dollars to farmers who are actually
ineligible for the program. This report should
horrify lawmakers, but it probably won’t.
(10) From 1995 to 2002, the United States Congress
doled out more than $114 billion to farmers. Why?
One misconception is that subsidies are
a boon to consumers because they lower food
prices. This ignores the fact that consumers are 
(15) also paying for these subsidies through taxes.
Because of inefficiencies in the program, we
taxpayers will pay more in taxes than we will ever
get back in lower corn or wheat prices.
In fact, farm subsidies are not even intended
(20) to reduce food prices significantly. When prices
are too low, farmers lose money. To prevent this
situation, Congress also pays farmers additional
“conservation subsidies” to leave their land fallow,
thereby lowering supply and boosting prices again.
(25) We’re taxed to lower prices, and then taxed to raise them again.
Another myth is that subsidies increase
exports, and thereby benefit the American
economy, by lowering the price of farm products
(30) and so making them more attractive to foreign
consumers. This ignores two realities. First,
farm subsidies transfer wealth from taxpayers
to foreign consumers just as efficiently as
they transfer wealth to domestic consumers.
(35) Second, farm subsidies are actually harming
American exporters. In March 2005, the World
Trade Organization ruled that American cotton
subsidies violated global free-trade rules, which
could lead to billions of dollars in retaliatory
(40) tariffs or penalties.
The worst misconception is that we need these
subsidies to save the small family farmer. Indeed,
according to a 2009 poll, about 77 percent of
Americans support giving subsidies to small family
(45) farms. But according to the Environmental Working
Group, 71 percent of farm subsidies go to the top
10 percent of beneficiaries, almost all of which are
large corporate farms. By subsidizing these rich
farmers, we actually make it much harder for the
(50) small family farmers to compete, not to mention
the millions of impoverished third world farmers
who rely on farming for their livelihood.
Rich corporate farmers are an enormously
powerful lobby in American politics. Agribusines
(55) and farm insurance lobbies pump nearly $100
million into political campaigns every year, and
the floodgates show no sign of closing. So don’t be
surprised if the GAO’s reports of mismanagement
and waste go unheeded. Politicians like their
(60) payouts almost as much as the big farmers and
their insurance companies do.

Passage 2
The critics of the U.S. farm subsidy program fail
to recognize just how vital these subsidies really
are. They are not as burdensome to American
(65) taxpayers as the critics claim, and indeed provide
important benefits. By protecting farmers from
damaging fluctuations in commodity prices due
to weather disasters or market disruptions, these
subsidies help sustain a vital American industry.
(70) At the same time, they protect consumers from
price spikes that can accompany steep drops in
crop inventories. Before price supports became
common in the 20th century, crop failures 
devastated the lives of farmers and consumers with
(75) horrifying frequency.
Opponents say that subsidies distort the
free market and create surpluses in supply. But
halting subsidies would allow regular shortfalls,
which are far more damaging. The year-to-year
(80) carryover of these surpluses protects farmers
from low prices and consumers from high prices.
Another misconception is that subsidies
only benefit the producers. In fact, they help
many related industries as well, including food
(85) processing, distribution, and marketing, chiefly
by helping to lower the cost of production. And,
of course, the consumers receive the benefit of
lower prices.
When assessing the costs and benefits of
(90) farm payments, it is important to compare these
subsidies to those of other industrialized nations.
American farmers receive an average of just 20% of
their incomes from subsidies, compared to 70% for
farmers from some other countries. The European
(95) Union spends about five times what the United
States spends on farm subsidies, amounting to
45% of the EU budget, compared to less than 1%
of the U.S. federal budget. Although the U.S. farm
subsidies programs are not perfect, they provide
(100) enormous benefits not only to farms but also
to associated industries employing millions of
people and to nearly every American consumer.

Q. If the author of Passage 2 were to use the data in the graph to support his main thesis, he would most likely mention

Detailed Solution for OneTime: Digital SAT Mock Test - 6 - Question 9

The thesis of Passage 2 is that farm subsidies in the United States are vital (line 63) and not as burdensome to American taxpayers as the critics claim (lines 64–65). Therefore, the author of Passage 2 would most likely cite evidence that the total cost of the subsidies program is declining.

OneTime: Digital SAT Mock Test - 6 - Question 10

Question are based on the following passage and supplementary material.
Passage 1 is adapted from Nicholas Heidorn, “The Enduring Political Illusion of Farm Subsidies.” ©2004 The Independent Institute. Originally Published August 18, 2004 in the San Francisco Chronicle. Passage 2 is ©2015 by Mark Anestis. Since 1922, the U.S. government has subsidized the agricultural industry by supporting the price of crops (commodity subsidies), paying farmers let their fields go fallow (conservation subsidies), helping farmers purchase crop insurance (crop insurance subsidies), and compensating farmers for uninsured losses due to disasters (disaster subsidies). The following passages discuss these programs.

Passage 1
Something is rotten down on the farm. A recent
General Accounting Office study found that the
U.S. farm subsidy program, a multibillion-dollar
system of direct payments to American farmers,
(5) uses administrators who are ill-trained and
poorly monitored, and who give away millions
of taxpayer dollars to farmers who are actually
ineligible for the program. This report should
horrify lawmakers, but it probably won’t.
(10) From 1995 to 2002, the United States Congress
doled out more than $114 billion to farmers. Why?
One misconception is that subsidies are
a boon to consumers because they lower food
prices. This ignores the fact that consumers are 
(15) also paying for these subsidies through taxes.
Because of inefficiencies in the program, we
taxpayers will pay more in taxes than we will ever
get back in lower corn or wheat prices.
In fact, farm subsidies are not even intended
(20) to reduce food prices significantly. When prices
are too low, farmers lose money. To prevent this
situation, Congress also pays farmers additional
“conservation subsidies” to leave their land fallow,
thereby lowering supply and boosting prices again.
(25) We’re taxed to lower prices, and then taxed to raise them again.
Another myth is that subsidies increase
exports, and thereby benefit the American
economy, by lowering the price of farm products
(30) and so making them more attractive to foreign
consumers. This ignores two realities. First,
farm subsidies transfer wealth from taxpayers
to foreign consumers just as efficiently as
they transfer wealth to domestic consumers.
(35) Second, farm subsidies are actually harming
American exporters. In March 2005, the World
Trade Organization ruled that American cotton
subsidies violated global free-trade rules, which
could lead to billions of dollars in retaliatory
(40) tariffs or penalties.
The worst misconception is that we need these
subsidies to save the small family farmer. Indeed,
according to a 2009 poll, about 77 percent of
Americans support giving subsidies to small family
(45) farms. But according to the Environmental Working
Group, 71 percent of farm subsidies go to the top
10 percent of beneficiaries, almost all of which are
large corporate farms. By subsidizing these rich
farmers, we actually make it much harder for the
(50) small family farmers to compete, not to mention
the millions of impoverished third world farmers
who rely on farming for their livelihood.
Rich corporate farmers are an enormously
powerful lobby in American politics. Agribusines
(55) and farm insurance lobbies pump nearly $100
million into political campaigns every year, and
the floodgates show no sign of closing. So don’t be
surprised if the GAO’s reports of mismanagement
and waste go unheeded. Politicians like their
(60) payouts almost as much as the big farmers and
their insurance companies do.

Passage 2
The critics of the U.S. farm subsidy program fail
to recognize just how vital these subsidies really
are. They are not as burdensome to American
(65) taxpayers as the critics claim, and indeed provide
important benefits. By protecting farmers from
damaging fluctuations in commodity prices due
to weather disasters or market disruptions, these
subsidies help sustain a vital American industry.
(70) At the same time, they protect consumers from
price spikes that can accompany steep drops in
crop inventories. Before price supports became
common in the 20th century, crop failures 
devastated the lives of farmers and consumers with
(75) horrifying frequency.
Opponents say that subsidies distort the
free market and create surpluses in supply. But
halting subsidies would allow regular shortfalls,
which are far more damaging. The year-to-year
(80) carryover of these surpluses protects farmers
from low prices and consumers from high prices.
Another misconception is that subsidies
only benefit the producers. In fact, they help
many related industries as well, including food
(85) processing, distribution, and marketing, chiefly
by helping to lower the cost of production. And,
of course, the consumers receive the benefit of
lower prices.
When assessing the costs and benefits of
(90) farm payments, it is important to compare these
subsidies to those of other industrialized nations.
American farmers receive an average of just 20% of
their incomes from subsidies, compared to 70% for
farmers from some other countries. The European
(95) Union spends about five times what the United
States spends on farm subsidies, amounting to
45% of the EU budget, compared to less than 1%
of the U.S. federal budget. Although the U.S. farm
subsidies programs are not perfect, they provide
(100) enormous benefits not only to farms but also
to associated industries employing millions of
people and to nearly every American consumer.

Q. The author of Passage 1 would most likely say that the “benefit” in line 87 is

Detailed Solution for OneTime: Digital SAT Mock Test - 6 - Question 10

The author of Passage 1 indicates that we taxpayers will pay more in taxes than we will ever get back in lower corn or wheat prices (lines 16–18), thereby indicating that the benefit of lower prices (lines 87–88) is offset by its costs.

OneTime: Digital SAT Mock Test - 6 - Question 11

Question based on the following passage and supplementary material.
Passage 1 is adapted from Nicholas Heidorn, “The Enduring Political Illusion of Farm Subsidies.” ©2004 The Independent Institute. Originally Published August 18, 2004 in the San Francisco Chronicle. Passage 2 is ©2015 by Mark Anestis. Since 1922, the U.S. government has subsidized the agricultural industry by supporting the price of crops (commodity subsidies), paying farmers let their fields go fallow (conservation subsidies), helping farmers purchase crop insurance (crop insurance subsidies), and compensating farmers for uninsured losses due to disasters (disaster subsidies). The following passages discuss these programs.

Passage 1
Something is rotten down on the farm. A recent
General Accounting Office study found that the
U.S. farm subsidy program, a multibillion-dollar
system of direct payments to American farmers,
(5) uses administrators who are ill-trained and
poorly monitored, and who give away millions
of taxpayer dollars to farmers who are actually
ineligible for the program. This report should
horrify lawmakers, but it probably won’t.
(10) From 1995 to 2002, the United States Congress
doled out more than $114 billion to farmers. Why?
One misconception is that subsidies are
a boon to consumers because they lower food
prices. This ignores the fact that consumers are 
(15) also paying for these subsidies through taxes.
Because of inefficiencies in the program, we
taxpayers will pay more in taxes than we will ever
get back in lower corn or wheat prices.
In fact, farm subsidies are not even intended
(20) to reduce food prices significantly. When prices
are too low, farmers lose money. To prevent this
situation, Congress also pays farmers additional
“conservation subsidies” to leave their land fallow,
thereby lowering supply and boosting prices again.
(25) We’re taxed to lower prices, and then taxed to raise them again.
Another myth is that subsidies increase
exports, and thereby benefit the American
economy, by lowering the price of farm products
(30) and so making them more attractive to foreign
consumers. This ignores two realities. First,
farm subsidies transfer wealth from taxpayers
to foreign consumers just as efficiently as
they transfer wealth to domestic consumers.
(35) Second, farm subsidies are actually harming
American exporters. In March 2005, the World
Trade Organization ruled that American cotton
subsidies violated global free-trade rules, which
could lead to billions of dollars in retaliatory
(40) tariffs or penalties.
The worst misconception is that we need these
subsidies to save the small family farmer. Indeed,
according to a 2009 poll, about 77 percent of
Americans support giving subsidies to small family
(45) farms. But according to the Environmental Working
Group, 71 percent of farm subsidies go to the top
10 percent of beneficiaries, almost all of which are
large corporate farms. By subsidizing these rich
farmers, we actually make it much harder for the
(50) small family farmers to compete, not to mention
the millions of impoverished third world farmers
who rely on farming for their livelihood.
Rich corporate farmers are an enormously
powerful lobby in American politics. Agribusines
(55) and farm insurance lobbies pump nearly $100
million into political campaigns every year, and
the floodgates show no sign of closing. So don’t be
surprised if the GAO’s reports of mismanagement
and waste go unheeded. Politicians like their
(60) payouts almost as much as the big farmers and
their insurance companies do.

Passage 2
The critics of the U.S. farm subsidy program fail
to recognize just how vital these subsidies really
are. They are not as burdensome to American
(65) taxpayers as the critics claim, and indeed provide
important benefits. By protecting farmers from
damaging fluctuations in commodity prices due
to weather disasters or market disruptions, these
subsidies help sustain a vital American industry.
(70) At the same time, they protect consumers from
price spikes that can accompany steep drops in
crop inventories. Before price supports became
common in the 20th century, crop failures 
devastated the lives of farmers and consumers with
(75) horrifying frequency.
Opponents say that subsidies distort the
free market and create surpluses in supply. But
halting subsidies would allow regular shortfalls,
which are far more damaging. The year-to-year
(80) carryover of these surpluses protects farmers
from low prices and consumers from high prices.
Another misconception is that subsidies
only benefit the producers. In fact, they help
many related industries as well, including food
(85) processing, distribution, and marketing, chiefly
by helping to lower the cost of production. And,
of course, the consumers receive the benefit of
lower prices.
When assessing the costs and benefits of
(90) farm payments, it is important to compare these
subsidies to those of other industrialized nations.
American farmers receive an average of just 20% of
their incomes from subsidies, compared to 70% for
farmers from some other countries. The European
(95) Union spends about five times what the United
States spends on farm subsidies, amounting to
45% of the EU budget, compared to less than 1%
of the U.S. federal budget. Although the U.S. farm
subsidies programs are not perfect, they provide
(100) enormous benefits not only to farms but also
to associated industries employing millions of
people and to nearly every American consumer.

Q. Unlike Passage 2, Passage 1 makes a direct appeal to the reader’s

Detailed Solution for OneTime: Digital SAT Mock Test - 6 - Question 11

The author of Passage 1 mentions that the U.S. farm subsidy programs use administrators who give away millions of taxpayer dollars to farmers who are actually ineligible for the program (lines 6–8) and are rife with inefficiencies (line 16) to make the argument that they are not worthwhile to taxpayers. This is an appeal to the reader’s distaste for ineptitude (incompetence).
Although this could also be seen as an appeal to the reader’s fiscal prudence (sense of responsibility), the author of Passage 2 makes the same kind of appeal when he indicates that these subsidies are vital (line 63) to preventing price spikes (line 71) and are not as burdensome to American taxpayers as the critics claim (lines 64–65). Since the question asks us to find an appeal that is NOT also found in Passage 2, choice (D) is incorrect.

OneTime: Digital SAT Mock Test - 6 - Question 12

Question based on the following passage and supplementary material.
Passage 1 is adapted from Nicholas Heidorn, “The Enduring Political Illusion of Farm Subsidies.” ©2004 The Independent Institute. Originally Published August 18, 2004 in the San Francisco Chronicle. Passage 2 is ©2015 by Mark Anestis. Since 1922, the U.S. government has subsidized the agricultural industry by supporting the price of crops (commodity subsidies), paying farmers let their fields go fallow (conservation subsidies), helping farmers purchase crop insurance (crop insurance subsidies), and compensating farmers for uninsured losses due to disasters (disaster subsidies). The following passages discuss these programs.

Passage 1
Something is rotten down on the farm. A recent
General Accounting Office study found that the
U.S. farm subsidy program, a multibillion-dollar
system of direct payments to American farmers,
(5) uses administrators who are ill-trained and
poorly monitored, and who give away millions
of taxpayer dollars to farmers who are actually
ineligible for the program. This report should
horrify lawmakers, but it probably won’t.
(10) From 1995 to 2002, the United States Congress
doled out more than $114 billion to farmers. Why?
One misconception is that subsidies are
a boon to consumers because they lower food
prices. This ignores the fact that consumers are 
(15) also paying for these subsidies through taxes.
Because of inefficiencies in the program, we
taxpayers will pay more in taxes than we will ever
get back in lower corn or wheat prices.
In fact, farm subsidies are not even intended
(20) to reduce food prices significantly. When prices
are too low, farmers lose money. To prevent this
situation, Congress also pays farmers additional
“conservation subsidies” to leave their land fallow,
thereby lowering supply and boosting prices again.
(25) We’re taxed to lower prices, and then taxed to raise them again.
Another myth is that subsidies increase
exports, and thereby benefit the American
economy, by lowering the price of farm products
(30) and so making them more attractive to foreign
consumers. This ignores two realities. First,
farm subsidies transfer wealth from taxpayers
to foreign consumers just as efficiently as
they transfer wealth to domestic consumers.
(35) Second, farm subsidies are actually harming
American exporters. In March 2005, the World
Trade Organization ruled that American cotton
subsidies violated global free-trade rules, which
could lead to billions of dollars in retaliatory
(40) tariffs or penalties.
The worst misconception is that we need these
subsidies to save the small family farmer. Indeed,
according to a 2009 poll, about 77 percent of
Americans support giving subsidies to small family
(45) farms. But according to the Environmental Working
Group, 71 percent of farm subsidies go to the top
10 percent of beneficiaries, almost all of which are
large corporate farms. By subsidizing these rich
farmers, we actually make it much harder for the
(50) small family farmers to compete, not to mention
the millions of impoverished third world farmers
who rely on farming for their livelihood.
Rich corporate farmers are an enormously
powerful lobby in American politics. Agribusines
(55) and farm insurance lobbies pump nearly $100
million into political campaigns every year, and
the floodgates show no sign of closing. So don’t be
surprised if the GAO’s reports of mismanagement
and waste go unheeded. Politicians like their
(60) payouts almost as much as the big farmers and
their insurance companies do.

Passage 2
The critics of the U.S. farm subsidy program fail
to recognize just how vital these subsidies really
are. They are not as burdensome to American
(65) taxpayers as the critics claim, and indeed provide
important benefits. By protecting farmers from
damaging fluctuations in commodity prices due
to weather disasters or market disruptions, these
subsidies help sustain a vital American industry.
(70) At the same time, they protect consumers from
price spikes that can accompany steep drops in
crop inventories. Before price supports became
common in the 20th century, crop failures 
devastated the lives of farmers and consumers with
(75) horrifying frequency.
Opponents say that subsidies distort the
free market and create surpluses in supply. But
halting subsidies would allow regular shortfalls,
which are far more damaging. The year-to-year
(80) carryover of these surpluses protects farmers
from low prices and consumers from high prices.
Another misconception is that subsidies
only benefit the producers. In fact, they help
many related industries as well, including food
(85) processing, distribution, and marketing, chiefly
by helping to lower the cost of production. And,
of course, the consumers receive the benefit of
lower prices.
When assessing the costs and benefits of
(90) farm payments, it is important to compare these
subsidies to those of other industrialized nations.
American farmers receive an average of just 20% of
their incomes from subsidies, compared to 70% for
farmers from some other countries. The European
(95) Union spends about five times what the United
States spends on farm subsidies, amounting to
45% of the EU budget, compared to less than 1%
of the U.S. federal budget. Although the U.S. farm
subsidies programs are not perfect, they provide
(100) enormous benefits not only to farms but also
to associated industries employing millions of
people and to nearly every American consumer.

Q. In line 57, the “floodgates” are controls against

Detailed Solution for OneTime: Digital SAT Mock Test - 6 - Question 12

When the author of Passage 1 states that Agribusiness and farm insurance lobbies pump nearly $100 million into political campaigns every year, and the floodgates show no sign of closing (lines 54–57), he suggests that there seem to be no controls against this unscrupulous (unethical) funding of political campaigns by those who benefit from the decisions of those politicians.

OneTime: Digital SAT Mock Test - 6 - Question 13

Question based on the following passage.
This passage is adapted from Marie Myung-Ok Lee, Somebody’s Daughter. ©2006 Beacon Press. The story is about a Korean-American girl adopted by an American family and raised in the Midwest.

When I was eight, they told me that my mother’s
death was preordained. She had been murdered.
One Sunday after service, our minister,
Reverend Jansen of the Lutheran Church of the
(5) Good Shepherd, bent down in a cloud of Aqua
Velva to explain. We had been learning in Sunday
school about Heaven and Hell, and in the middle
of class I had fallen into a panic, wondering how
I would recognize my Korean mother when I saw
(10) her in Heaven—or in Hell, if perhaps she and I
both sinned too much.
Not to worry, I was told.
“God called your Korean parents home so
that you could become the daughter of your
(15) mother and father,” he said, his eyes sliding
sidewise, for just a second. His breath smelled
vaguely of toast.
“It was all part of His plan—you see how much
your mommy and daddy love you? When the time
(20) comes, if you’re a very good girl, you, your mommy,
daddy, and your sister, Amanda—the whole
Thorson family—will be in heaven together, thanks
to the Lord’s wonderful and mysterious ways.”
“That’s why we named you Sarah,” Christine
(25) and Ken added. “Because it means ‘God’s
precious treasure.’”
God kills, I thought then. The same God who
brought us Christmas and the Easter Bunny—he
murdered my mother.
(30) Shortly after that Sunday, I brought up
my Korean mother again, asking about the car
accident, how it happened, exactly—was it like
Phil Haag’s father, who fell asleep at the wheel? Or
like our plumber’s teenage son who drove into a
(35) semi head-on?
“Sarah,” Christine said patiently, looking
up from the chopping board, where she was
slicing carrot discs for pot roast. “We really knew
nothing about her. I’m your mommy. Let’s not 
(40) talk about this any more, it makes me sad.” She
made little crying motions, pretending to wipe
away tears, the same thing she did when I was
bad, to show how I had disappointed her.
I grew up in a house in which Korea had
(45) always been the oddly charged word, never to be
mentioned in connection to me, the same way
we never said “Uncle Henry” and “alcoholic” in
the same sentence. It was almost as if Ken and
Christine thought I needed to be protected from
(50) it, the way small children need to be protected
from boors itching to tell them that Santa Claus
is not real.The ban on Korea extended even to
the aforementioned Uncle Henry, who was then
deprived of his war stories at our Memorial Day
(55) cookouts. Although he proudly wore his felt VFW
hat with its flurry of pins, including ones from his
tour “overseas,” Christine or Ken would quietly
slip him some of his favorite Pabst or Schlitz, and
in return he’d set up residence in the lawn chair
(60) at the far corner of our yard, away from everyone.
Somewhere back in the fuzzy clot of my
teens (now, I’m at the worldly-wise age of almost-
twenty), the ’88 Summer Olympics were held
in Seoul. We couldn’t buck the Thorson family
(65) tradition of watching absolutely everything (that
winter we’d raptly watched curling, for God’s
sakes!). But I was aware that pains were taken
to modulate voices, vocal cords twisted to an
excruciating, studied casualness until Korea
(70) came out “Korea,” exactly the same way we’d say
“Russia” or “Carl Lewis” or “Flo-Jo.”
Then Bryant Gumbel invaded our living room
with his special segment on how Korea, one of the
four “Little Tiger” economic miracle countries,
(75) was so enterprising that it had even made an
export product out of its babies. Since the Korean
War, more than a hundred thousand children,
Made-in-Korea stamped on their foreheads, had
left the country, their adoption fees fattening the
(80) government coffers.
Top that, Singapore! Gumbel’s cheery smirk
seemed to say.
“Well, Sarah’s really American, not Kor—”
Amanda began, until the look on Christine’s
(85) face—despairing, fierce—stopped her.
We invent what becomes us.

Q. The narrator characterizes Reverend Jansen primarily as

Detailed Solution for OneTime: Digital SAT Mock Test - 6 - Question 13

The narrator says that Reverend Jansen bent down in a cloud of Aqua Velva (lines 5–6) and told her not to worry (line 12). He then describes to the narrator why God called [her] Korean parents home (line 13). All of these descriptions work together to portray someone who is acting in a condescending and patronizing manner to a young child.

OneTime: Digital SAT Mock Test - 6 - Question 14

Question based on the following passage.
This passage is adapted from Marie Myung-Ok Lee, Somebody’s Daughter. ©2006 Beacon Press. The story is about a Korean-American girl adopted by an American family and raised in the Midwest.

When I was eight, they told me that my mother’s
death was preordained. She had been murdered.
One Sunday after service, our minister,
Reverend Jansen of the Lutheran Church of the
(5) Good Shepherd, bent down in a cloud of Aqua
Velva to explain. We had been learning in Sunday
school about Heaven and Hell, and in the middle
of class I had fallen into a panic, wondering how
I would recognize my Korean mother when I saw
(10) her in Heaven—or in Hell, if perhaps she and I
both sinned too much.
Not to worry, I was told.
“God called your Korean parents home so
that you could become the daughter of your
(15) mother and father,” he said, his eyes sliding
sidewise, for just a second. His breath smelled
vaguely of toast.
“It was all part of His plan—you see how much
your mommy and daddy love you? When the time
(20) comes, if you’re a very good girl, you, your mommy,
daddy, and your sister, Amanda—the whole
Thorson family—will be in heaven together, thanks
to the Lord’s wonderful and mysterious ways.”
“That’s why we named you Sarah,” Christine
(25) and Ken added. “Because it means ‘God’s
precious treasure.’”
God kills, I thought then. The same God who
brought us Christmas and the Easter Bunny—he
murdered my mother.
(30) Shortly after that Sunday, I brought up
my Korean mother again, asking about the car
accident, how it happened, exactly—was it like
Phil Haag’s father, who fell asleep at the wheel? Or
like our plumber’s teenage son who drove into a
(35) semi head-on?
“Sarah,” Christine said patiently, looking
up from the chopping board, where she was
slicing carrot discs for pot roast. “We really knew
nothing about her. I’m your mommy. Let’s not 
(40) talk about this any more, it makes me sad.” She
made little crying motions, pretending to wipe
away tears, the same thing she did when I was
bad, to show how I had disappointed her.
I grew up in a house in which Korea had
(45) always been the oddly charged word, never to be
mentioned in connection to me, the same way
we never said “Uncle Henry” and “alcoholic” in
the same sentence. It was almost as if Ken and
Christine thought I needed to be protected from
(50) it, the way small children need to be protected
from boors itching to tell them that Santa Claus
is not real.The ban on Korea extended even to
the aforementioned Uncle Henry, who was then
deprived of his war stories at our Memorial Day
(55) cookouts. Although he proudly wore his felt VFW
hat with its flurry of pins, including ones from his
tour “overseas,” Christine or Ken would quietly
slip him some of his favorite Pabst or Schlitz, and
in return he’d set up residence in the lawn chair
(60) at the far corner of our yard, away from everyone.
Somewhere back in the fuzzy clot of my
teens (now, I’m at the worldly-wise age of almost-
twenty), the ’88 Summer Olympics were held
in Seoul. We couldn’t buck the Thorson family
(65) tradition of watching absolutely everything (that
winter we’d raptly watched curling, for God’s
sakes!). But I was aware that pains were taken
to modulate voices, vocal cords twisted to an
excruciating, studied casualness until Korea
(70) came out “Korea,” exactly the same way we’d say
“Russia” or “Carl Lewis” or “Flo-Jo.”
Then Bryant Gumbel invaded our living room
with his special segment on how Korea, one of the
four “Little Tiger” economic miracle countries,
(75) was so enterprising that it had even made an
export product out of its babies. Since the Korean
War, more than a hundred thousand children,
Made-in-Korea stamped on their foreheads, had
left the country, their adoption fees fattening the
(80) government coffers.
Top that, Singapore! Gumbel’s cheery smirk
seemed to say.
“Well, Sarah’s really American, not Kor—”
Amanda began, until the look on Christine’s
(85) face—despairing, fierce—stopped her.
We invent what becomes us.

Q. The narrator’s statement that her mother “had been murdered” (line 2) is best taken to mean that

Detailed Solution for OneTime: Digital SAT Mock Test - 6 - Question 14

The statement that the narrator’s mother had been murdered (line 2) is later explained to refer to the narrator’s interpretation of the fact that she was told that “God called [her] Korean parents home” (line 13) and that “It was all part of His plan” (line 18), in other words, her death was deliberate. At first, choice (D) may seem plausible, because in lines 36–43, Sarah’s mother does not want to talk about Sarah’s biological mother. However, the passage makes it clear that the narrator attributed the “murder” to a divine plan (God kills, I thought then, l i ne 27) rat her t ha n to a ny i ntent ion of her adoptive family.

OneTime: Digital SAT Mock Test - 6 - Question 15

Question based on the following passage.
This passage is adapted from Marie Myung-Ok Lee, Somebody’s Daughter. ©2006 Beacon Press. The story is about a Korean-American girl adopted by an American family and raised in the Midwest.

When I was eight, they told me that my mother’s
death was preordained. She had been murdered.
One Sunday after service, our minister,
Reverend Jansen of the Lutheran Church of the
(5) Good Shepherd, bent down in a cloud of Aqua
Velva to explain. We had been learning in Sunday
school about Heaven and Hell, and in the middle
of class I had fallen into a panic, wondering how
I would recognize my Korean mother when I saw
(10) her in Heaven—or in Hell, if perhaps she and I
both sinned too much.
Not to worry, I was told.
“God called your Korean parents home so
that you could become the daughter of your
(15) mother and father,” he said, his eyes sliding
sidewise, for just a second. His breath smelled
vaguely of toast.
“It was all part of His plan—you see how much
your mommy and daddy love you? When the time
(20) comes, if you’re a very good girl, you, your mommy,
daddy, and your sister, Amanda—the whole
Thorson family—will be in heaven together, thanks
to the Lord’s wonderful and mysterious ways.”
“That’s why we named you Sarah,” Christine
(25) and Ken added. “Because it means ‘God’s
precious treasure.’”
God kills, I thought then. The same God who
brought us Christmas and the Easter Bunny—he
murdered my mother.
(30) Shortly after that Sunday, I brought up
my Korean mother again, asking about the car
accident, how it happened, exactly—was it like
Phil Haag’s father, who fell asleep at the wheel? Or
like our plumber’s teenage son who drove into a
(35) semi head-on?
“Sarah,” Christine said patiently, looking
up from the chopping board, where she was
slicing carrot discs for pot roast. “We really knew
nothing about her. I’m your mommy. Let’s not 
(40) talk about this any more, it makes me sad.” She
made little crying motions, pretending to wipe
away tears, the same thing she did when I was
bad, to show how I had disappointed her.
I grew up in a house in which Korea had
(45) always been the oddly charged word, never to be
mentioned in connection to me, the same way
we never said “Uncle Henry” and “alcoholic” in
the same sentence. It was almost as if Ken and
Christine thought I needed to be protected from
(50) it, the way small children need to be protected
from boors itching to tell them that Santa Claus
is not real.The ban on Korea extended even to
the aforementioned Uncle Henry, who was then
deprived of his war stories at our Memorial Day
(55) cookouts. Although he proudly wore his felt VFW
hat with its flurry of pins, including ones from his
tour “overseas,” Christine or Ken would quietly
slip him some of his favorite Pabst or Schlitz, and
in return he’d set up residence in the lawn chair
(60) at the far corner of our yard, away from everyone.
Somewhere back in the fuzzy clot of my
teens (now, I’m at the worldly-wise age of almost-
twenty), the ’88 Summer Olympics were held
in Seoul. We couldn’t buck the Thorson family
(65) tradition of watching absolutely everything (that
winter we’d raptly watched curling, for God’s
sakes!). But I was aware that pains were taken
to modulate voices, vocal cords twisted to an
excruciating, studied casualness until Korea
(70) came out “Korea,” exactly the same way we’d say
“Russia” or “Carl Lewis” or “Flo-Jo.”
Then Bryant Gumbel invaded our living room
with his special segment on how Korea, one of the
four “Little Tiger” economic miracle countries,
(75) was so enterprising that it had even made an
export product out of its babies. Since the Korean
War, more than a hundred thousand children,
Made-in-Korea stamped on their foreheads, had
left the country, their adoption fees fattening the
(80) government coffers.
Top that, Singapore! Gumbel’s cheery smirk
seemed to say.
“Well, Sarah’s really American, not Kor—”
Amanda began, until the look on Christine’s
(85) face—despairing, fierce—stopped her.
We invent what becomes us.

Q. The narrator’s description of the reverend’s “eyes” and “breath” in lines 15–16 primarily convey a sense of

Detailed Solution for OneTime: Digital SAT Mock Test - 6 - Question 15

The description of Reverend Jansen’s eyes and breath in lines 13–17 indicates that he is somewhat emotionally detached (his eyes sliding sideways, lines 15–16) and that Sarah is likewise emotionally detached from him and his profound claims, instead distracted by his breath that smelled vaguely of toast (lines 16–17). These descriptions surprise us, because they are so incongruent with the expectation of respect for and contemplation of the reverend’s deep spiritual pronouncements.

OneTime: Digital SAT Mock Test - 6 - Question 16

Question based on the following passage.
This passage is adapted from Marie Myung-Ok Lee, Somebody’s Daughter. ©2006 Beacon Press. The story is about a Korean-American girl adopted by an American family and raised in the Midwest.

When I was eight, they told me that my mother’s
death was preordained. She had been murdered.
One Sunday after service, our minister,
Reverend Jansen of the Lutheran Church of the
(5) Good Shepherd, bent down in a cloud of Aqua
Velva to explain. We had been learning in Sunday
school about Heaven and Hell, and in the middle
of class I had fallen into a panic, wondering how
I would recognize my Korean mother when I saw
(10) her in Heaven—or in Hell, if perhaps she and I
both sinned too much.
Not to worry, I was told.
“God called your Korean parents home so
that you could become the daughter of your
(15) mother and father,” he said, his eyes sliding
sidewise, for just a second. His breath smelled
vaguely of toast.
“It was all part of His plan—you see how much
your mommy and daddy love you? When the time
(20) comes, if you’re a very good girl, you, your mommy,
daddy, and your sister, Amanda—the whole
Thorson family—will be in heaven together, thanks
to the Lord’s wonderful and mysterious ways.”
“That’s why we named you Sarah,” Christine
(25) and Ken added. “Because it means ‘God’s
precious treasure.’”
God kills, I thought then. The same God who
brought us Christmas and the Easter Bunny—he
murdered my mother.
(30) Shortly after that Sunday, I brought up
my Korean mother again, asking about the car
accident, how it happened, exactly—was it like
Phil Haag’s father, who fell asleep at the wheel? Or
like our plumber’s teenage son who drove into a
(35) semi head-on?
“Sarah,” Christine said patiently, looking
up from the chopping board, where she was
slicing carrot discs for pot roast. “We really knew
nothing about her. I’m your mommy. Let’s not 
(40) talk about this any more, it makes me sad.” She
made little crying motions, pretending to wipe
away tears, the same thing she did when I was
bad, to show how I had disappointed her.
I grew up in a house in which Korea had
(45) always been the oddly charged word, never to be
mentioned in connection to me, the same way
we never said “Uncle Henry” and “alcoholic” in
the same sentence. It was almost as if Ken and
Christine thought I needed to be protected from
(50) it, the way small children need to be protected
from boors itching to tell them that Santa Claus
is not real.The ban on Korea extended even to
the aforementioned Uncle Henry, who was then
deprived of his war stories at our Memorial Day
(55) cookouts. Although he proudly wore his felt VFW
hat with its flurry of pins, including ones from his
tour “overseas,” Christine or Ken would quietly
slip him some of his favorite Pabst or Schlitz, and
in return he’d set up residence in the lawn chair
(60) at the far corner of our yard, away from everyone.
Somewhere back in the fuzzy clot of my
teens (now, I’m at the worldly-wise age of almost-
twenty), the ’88 Summer Olympics were held
in Seoul. We couldn’t buck the Thorson family
(65) tradition of watching absolutely everything (that
winter we’d raptly watched curling, for God’s
sakes!). But I was aware that pains were taken
to modulate voices, vocal cords twisted to an
excruciating, studied casualness until Korea
(70) came out “Korea,” exactly the same way we’d say
“Russia” or “Carl Lewis” or “Flo-Jo.”
Then Bryant Gumbel invaded our living room
with his special segment on how Korea, one of the
four “Little Tiger” economic miracle countries,
(75) was so enterprising that it had even made an
export product out of its babies. Since the Korean
War, more than a hundred thousand children,
Made-in-Korea stamped on their foreheads, had
left the country, their adoption fees fattening the
(80) government coffers.
Top that, Singapore! Gumbel’s cheery smirk
seemed to say.
“Well, Sarah’s really American, not Kor—”
Amanda began, until the look on Christine’s
(85) face—despairing, fierce—stopped her.
We invent what becomes us.

Q. Christine believes that Sarah’s ethnicity is

Detailed Solution for OneTime: Digital SAT Mock Test - 6 - Question 16

The narrator has Korean heritage, yet she grew up in a house in which Korea had always been the oddly charged word, never to be mentioned in connection with [Sarah], the same way [they] never said “Uncle Henry” and “alcoholic” in the same sentence (lines 44–47). The narrator’s mother, Christine, thought [Sarah] needed to protected from (line 49) her ethnicity. In other words, she regarded Sarah’s ethnicity as an unfortunate fact.

OneTime: Digital SAT Mock Test - 6 - Question 17

Question based on the following passage.
This passage is adapted from Marie Myung-Ok Lee, Somebody’s Daughter. ©2006 Beacon Press. The story is about a Korean-American girl adopted by an American family and raised in the Midwest.

When I was eight, they told me that my mother’s
death was preordained. She had been murdered.
One Sunday after service, our minister,
Reverend Jansen of the Lutheran Church of the
(5) Good Shepherd, bent down in a cloud of Aqua
Velva to explain. We had been learning in Sunday
school about Heaven and Hell, and in the middle
of class I had fallen into a panic, wondering how
I would recognize my Korean mother when I saw
(10) her in Heaven—or in Hell, if perhaps she and I
both sinned too much.
Not to worry, I was told.
“God called your Korean parents home so
that you could become the daughter of your
(15) mother and father,” he said, his eyes sliding
sidewise, for just a second. His breath smelled
vaguely of toast.
“It was all part of His plan—you see how much
your mommy and daddy love you? When the time
(20) comes, if you’re a very good girl, you, your mommy,
daddy, and your sister, Amanda—the whole
Thorson family—will be in heaven together, thanks
to the Lord’s wonderful and mysterious ways.”
“That’s why we named you Sarah,” Christine
(25) and Ken added. “Because it means ‘God’s
precious treasure.’”
God kills, I thought then. The same God who
brought us Christmas and the Easter Bunny—he
murdered my mother.
(30) Shortly after that Sunday, I brought up
my Korean mother again, asking about the car
accident, how it happened, exactly—was it like
Phil Haag’s father, who fell asleep at the wheel? Or
like our plumber’s teenage son who drove into a
(35) semi head-on?
“Sarah,” Christine said patiently, looking
up from the chopping board, where she was
slicing carrot discs for pot roast. “We really knew
nothing about her. I’m your mommy. Let’s not 
(40) talk about this any more, it makes me sad.” She
made little crying motions, pretending to wipe
away tears, the same thing she did when I was
bad, to show how I had disappointed her.
I grew up in a house in which Korea had
(45) always been the oddly charged word, never to be
mentioned in connection to me, the same way
we never said “Uncle Henry” and “alcoholic” in
the same sentence. It was almost as if Ken and
Christine thought I needed to be protected from
(50) it, the way small children need to be protected
from boors itching to tell them that Santa Claus
is not real.The ban on Korea extended even to
the aforementioned Uncle Henry, who was then
deprived of his war stories at our Memorial Day
(55) cookouts. Although he proudly wore his felt VFW
hat with its flurry of pins, including ones from his
tour “overseas,” Christine or Ken would quietly
slip him some of his favorite Pabst or Schlitz, and
in return he’d set up residence in the lawn chair
(60) at the far corner of our yard, away from everyone.
Somewhere back in the fuzzy clot of my
teens (now, I’m at the worldly-wise age of almost-
twenty), the ’88 Summer Olympics were held
in Seoul. We couldn’t buck the Thorson family
(65) tradition of watching absolutely everything (that
winter we’d raptly watched curling, for God’s
sakes!). But I was aware that pains were taken
to modulate voices, vocal cords twisted to an
excruciating, studied casualness until Korea
(70) came out “Korea,” exactly the same way we’d say
“Russia” or “Carl Lewis” or “Flo-Jo.”
Then Bryant Gumbel invaded our living room
with his special segment on how Korea, one of the
four “Little Tiger” economic miracle countries,
(75) was so enterprising that it had even made an
export product out of its babies. Since the Korean
War, more than a hundred thousand children,
Made-in-Korea stamped on their foreheads, had
left the country, their adoption fees fattening the
(80) government coffers.
Top that, Singapore! Gumbel’s cheery smirk
seemed to say.
“Well, Sarah’s really American, not Kor—”
Amanda began, until the look on Christine’s
(85) face—despairing, fierce—stopped her.
We invent what becomes us.

Q. Which choice provides the best evidence for the answer to the previous question?

Detailed Solution for OneTime: Digital SAT Mock Test - 6 - Question 17

As the explanation to question 16 makes clear, the best evidence for the previous answer is in lines 48–52

OneTime: Digital SAT Mock Test - 6 - Question 18

Question based on the following passage.
This passage is adapted from Marie Myung-Ok Lee, Somebody’s Daughter. ©2006 Beacon Press. The story is about a Korean-American girl adopted by an American family and raised in the Midwest.

When I was eight, they told me that my mother’s
death was preordained. She had been murdered.
One Sunday after service, our minister,
Reverend Jansen of the Lutheran Church of the
(5) Good Shepherd, bent down in a cloud of Aqua
Velva to explain. We had been learning in Sunday
school about Heaven and Hell, and in the middle
of class I had fallen into a panic, wondering how
I would recognize my Korean mother when I saw
(10) her in Heaven—or in Hell, if perhaps she and I
both sinned too much.
Not to worry, I was told.
“God called your Korean parents home so
that you could become the daughter of your
(15) mother and father,” he said, his eyes sliding
sidewise, for just a second. His breath smelled
vaguely of toast.
“It was all part of His plan—you see how much
your mommy and daddy love you? When the time
(20) comes, if you’re a very good girl, you, your mommy,
daddy, and your sister, Amanda—the whole
Thorson family—will be in heaven together, thanks
to the Lord’s wonderful and mysterious ways.”
“That’s why we named you Sarah,” Christine
(25) and Ken added. “Because it means ‘God’s
precious treasure.’”
God kills, I thought then. The same God who
brought us Christmas and the Easter Bunny—he
murdered my mother.
(30) Shortly after that Sunday, I brought up
my Korean mother again, asking about the car
accident, how it happened, exactly—was it like
Phil Haag’s father, who fell asleep at the wheel? Or
like our plumber’s teenage son who drove into a
(35) semi head-on?
“Sarah,” Christine said patiently, looking
up from the chopping board, where she was
slicing carrot discs for pot roast. “We really knew
nothing about her. I’m your mommy. Let’s not 
(40) talk about this any more, it makes me sad.” She
made little crying motions, pretending to wipe
away tears, the same thing she did when I was
bad, to show how I had disappointed her.
I grew up in a house in which Korea had
(45) always been the oddly charged word, never to be
mentioned in connection to me, the same way
we never said “Uncle Henry” and “alcoholic” in
the same sentence. It was almost as if Ken and
Christine thought I needed to be protected from
(50) it, the way small children need to be protected
from boors itching to tell them that Santa Claus
is not real.The ban on Korea extended even to
the aforementioned Uncle Henry, who was then
deprived of his war stories at our Memorial Day
(55) cookouts. Although he proudly wore his felt VFW
hat with its flurry of pins, including ones from his
tour “overseas,” Christine or Ken would quietly
slip him some of his favorite Pabst or Schlitz, and
in return he’d set up residence in the lawn chair
(60) at the far corner of our yard, away from everyone.
Somewhere back in the fuzzy clot of my
teens (now, I’m at the worldly-wise age of almost-
twenty), the ’88 Summer Olympics were held
in Seoul. We couldn’t buck the Thorson family
(65) tradition of watching absolutely everything (that
winter we’d raptly watched curling, for God’s
sakes!). But I was aware that pains were taken
to modulate voices, vocal cords twisted to an
excruciating, studied casualness until Korea
(70) came out “Korea,” exactly the same way we’d say
“Russia” or “Carl Lewis” or “Flo-Jo.”
Then Bryant Gumbel invaded our living room
with his special segment on how Korea, one of the
four “Little Tiger” economic miracle countries,
(75) was so enterprising that it had even made an
export product out of its babies. Since the Korean
War, more than a hundred thousand children,
Made-in-Korea stamped on their foreheads, had
left the country, their adoption fees fattening the
(80) government coffers.
Top that, Singapore! Gumbel’s cheery smirk
seemed to say.
“Well, Sarah’s really American, not Kor—”
Amanda began, until the look on Christine’s
(85) face—despairing, fierce—stopped her.
We invent what becomes us.

Q. Lines 27–29 (“God kills . . . my mother”) are striking for their use of

Detailed Solution for OneTime: Digital SAT Mock Test - 6 - Question 18

The contrast between murder and Christmas and the Easter Bunny (line 28) is a classic example of juxtaposition, the act of placing together two images with highly contrasting effects.

OneTime: Digital SAT Mock Test - 6 - Question 19

Question based on the following passage.
This passage is adapted from Marie Myung-Ok Lee, Somebody’s Daughter. ©2006 Beacon Press. The story is about a Korean-American girl adopted by an American family and raised in the Midwest.

When I was eight, they told me that my mother’s
death was preordained. She had been murdered.
One Sunday after service, our minister,
Reverend Jansen of the Lutheran Church of the
(5) Good Shepherd, bent down in a cloud of Aqua
Velva to explain. We had been learning in Sunday
school about Heaven and Hell, and in the middle
of class I had fallen into a panic, wondering how
I would recognize my Korean mother when I saw
(10) her in Heaven—or in Hell, if perhaps she and I
both sinned too much.
Not to worry, I was told.
“God called your Korean parents home so
that you could become the daughter of your
(15) mother and father,” he said, his eyes sliding
sidewise, for just a second. His breath smelled
vaguely of toast.
“It was all part of His plan—you see how much
your mommy and daddy love you? When the time
(20) comes, if you’re a very good girl, you, your mommy,
daddy, and your sister, Amanda—the whole
Thorson family—will be in heaven together, thanks
to the Lord’s wonderful and mysterious ways.”
“That’s why we named you Sarah,” Christine
(25) and Ken added. “Because it means ‘God’s
precious treasure.’”
God kills, I thought then. The same God who
brought us Christmas and the Easter Bunny—he
murdered my mother.
(30) Shortly after that Sunday, I brought up
my Korean mother again, asking about the car
accident, how it happened, exactly—was it like
Phil Haag’s father, who fell asleep at the wheel? Or
like our plumber’s teenage son who drove into a
(35) semi head-on?
“Sarah,” Christine said patiently, looking
up from the chopping board, where she was
slicing carrot discs for pot roast. “We really knew
nothing about her. I’m your mommy. Let’s not 
(40) talk about this any more, it makes me sad.” She
made little crying motions, pretending to wipe
away tears, the same thing she did when I was
bad, to show how I had disappointed her.
I grew up in a house in which Korea had
(45) always been the oddly charged word, never to be
mentioned in connection to me, the same way
we never said “Uncle Henry” and “alcoholic” in
the same sentence. It was almost as if Ken and
Christine thought I needed to be protected from
(50) it, the way small children need to be protected
from boors itching to tell them that Santa Claus
is not real.The ban on Korea extended even to
the aforementioned Uncle Henry, who was then
deprived of his war stories at our Memorial Day
(55) cookouts. Although he proudly wore his felt VFW
hat with its flurry of pins, including ones from his
tour “overseas,” Christine or Ken would quietly
slip him some of his favorite Pabst or Schlitz, and
in return he’d set up residence in the lawn chair
(60) at the far corner of our yard, away from everyone.
Somewhere back in the fuzzy clot of my
teens (now, I’m at the worldly-wise age of almost-
twenty), the ’88 Summer Olympics were held
in Seoul. We couldn’t buck the Thorson family
(65) tradition of watching absolutely everything (that
winter we’d raptly watched curling, for God’s
sakes!). But I was aware that pains were taken
to modulate voices, vocal cords twisted to an
excruciating, studied casualness until Korea
(70) came out “Korea,” exactly the same way we’d say
“Russia” or “Carl Lewis” or “Flo-Jo.”
Then Bryant Gumbel invaded our living room
with his special segment on how Korea, one of the
four “Little Tiger” economic miracle countries,
(75) was so enterprising that it had even made an
export product out of its babies. Since the Korean
War, more than a hundred thousand children,
Made-in-Korea stamped on their foreheads, had
left the country, their adoption fees fattening the
(80) government coffers.
Top that, Singapore! Gumbel’s cheery smirk
seemed to say.
“Well, Sarah’s really American, not Kor—”
Amanda began, until the look on Christine’s
(85) face—despairing, fierce—stopped her.
We invent what becomes us.

Q. Lines 36–44 chiefly describe Christine’s

Detailed Solution for OneTime: Digital SAT Mock Test - 6 - Question 19

The paragraph states that Christine begins her reply patiently (line 36), which might suggest that she is demonstrating motherly sympathy. However, sympathy means “a feeling of common understanding,” and the rest of Christine’s reply suggests that she is disappointed (line 43) with Sarah rather than sympathetic with her. The point of the paragraph is that Christine is not emotionally ready (it makes me sad, line 40) to discuss something that her eight-year-old adopted daughter clearly wants to discuss, that is, she is emotionally immature.

OneTime: Digital SAT Mock Test - 6 - Question 20

Question based on the following passage.
This passage is adapted from Marie Myung-Ok Lee, Somebody’s Daughter. ©2006 Beacon Press. The story is about a Korean-American girl adopted by an American family and raised in the Midwest.

When I was eight, they told me that my mother’s
death was preordained. She had been murdered.
One Sunday after service, our minister,
Reverend Jansen of the Lutheran Church of the
(5) Good Shepherd, bent down in a cloud of Aqua
Velva to explain. We had been learning in Sunday
school about Heaven and Hell, and in the middle
of class I had fallen into a panic, wondering how
I would recognize my Korean mother when I saw
(10) her in Heaven—or in Hell, if perhaps she and I
both sinned too much.
Not to worry, I was told.
“God called your Korean parents home so
that you could become the daughter of your
(15) mother and father,” he said, his eyes sliding
sidewise, for just a second. His breath smelled
vaguely of toast.
“It was all part of His plan—you see how much
your mommy and daddy love you? When the time
(20) comes, if you’re a very good girl, you, your mommy,
daddy, and your sister, Amanda—the whole
Thorson family—will be in heaven together, thanks
to the Lord’s wonderful and mysterious ways.”
“That’s why we named you Sarah,” Christine
(25) and Ken added. “Because it means ‘God’s
precious treasure.’”
God kills, I thought then. The same God who
brought us Christmas and the Easter Bunny—he
murdered my mother.
(30) Shortly after that Sunday, I brought up
my Korean mother again, asking about the car
accident, how it happened, exactly—was it like
Phil Haag’s father, who fell asleep at the wheel? Or
like our plumber’s teenage son who drove into a
(35) semi head-on?
“Sarah,” Christine said patiently, looking
up from the chopping board, where she was
slicing carrot discs for pot roast. “We really knew
nothing about her. I’m your mommy. Let’s not 
(40) talk about this any more, it makes me sad.” She
made little crying motions, pretending to wipe
away tears, the same thing she did when I was
bad, to show how I had disappointed her.
I grew up in a house in which Korea had
(45) always been the oddly charged word, never to be
mentioned in connection to me, the same way
we never said “Uncle Henry” and “alcoholic” in
the same sentence. It was almost as if Ken and
Christine thought I needed to be protected from
(50) it, the way small children need to be protected
from boors itching to tell them that Santa Claus
is not real.The ban on Korea extended even to
the aforementioned Uncle Henry, who was then
deprived of his war stories at our Memorial Day
(55) cookouts. Although he proudly wore his felt VFW
hat with its flurry of pins, including ones from his
tour “overseas,” Christine or Ken would quietly
slip him some of his favorite Pabst or Schlitz, and
in return he’d set up residence in the lawn chair
(60) at the far corner of our yard, away from everyone.
Somewhere back in the fuzzy clot of my
teens (now, I’m at the worldly-wise age of almost-
twenty), the ’88 Summer Olympics were held
in Seoul. We couldn’t buck the Thorson family
(65) tradition of watching absolutely everything (that
winter we’d raptly watched curling, for God’s
sakes!). But I was aware that pains were taken
to modulate voices, vocal cords twisted to an
excruciating, studied casualness until Korea
(70) came out “Korea,” exactly the same way we’d say
“Russia” or “Carl Lewis” or “Flo-Jo.”
Then Bryant Gumbel invaded our living room
with his special segment on how Korea, one of the
four “Little Tiger” economic miracle countries,
(75) was so enterprising that it had even made an
export product out of its babies. Since the Korean
War, more than a hundred thousand children,
Made-in-Korea stamped on their foreheads, had
left the country, their adoption fees fattening the
(80) government coffers.
Top that, Singapore! Gumbel’s cheery smirk
seemed to say.
“Well, Sarah’s really American, not Kor—”
Amanda began, until the look on Christine’s
(85) face—despairing, fierce—stopped her.
We invent what becomes us.

Q. In line 45, “charged” most nearly means

Detailed Solution for OneTime: Digital SAT Mock Test - 6 - Question 20

When the narrator states that Korea had always been the oddly charged word (lines 44–45), she means that it was a word that was never to be mentioned (lines 45–46), because it was associated with potentially negative feelings. That is, it was an emotionally loaded word.

OneTime: Digital SAT Mock Test - 6 - Question 21

Question based on the following passage.
This passage is adapted from Marie Myung-Ok Lee, Somebody’s Daughter. ©2006 Beacon Press. The story is about a Korean-American girl adopted by an American family and raised in the Midwest.

When I was eight, they told me that my mother’s
death was preordained. She had been murdered.
One Sunday after service, our minister,
Reverend Jansen of the Lutheran Church of the
(5) Good Shepherd, bent down in a cloud of Aqua
Velva to explain. We had been learning in Sunday
school about Heaven and Hell, and in the middle
of class I had fallen into a panic, wondering how
I would recognize my Korean mother when I saw
(10) her in Heaven—or in Hell, if perhaps she and I
both sinned too much.
Not to worry, I was told.
“God called your Korean parents home so
that you could become the daughter of your
(15) mother and father,” he said, his eyes sliding
sidewise, for just a second. His breath smelled
vaguely of toast.
“It was all part of His plan—you see how much
your mommy and daddy love you? When the time
(20) comes, if you’re a very good girl, you, your mommy,
daddy, and your sister, Amanda—the whole
Thorson family—will be in heaven together, thanks
to the Lord’s wonderful and mysterious ways.”
“That’s why we named you Sarah,” Christine
(25) and Ken added. “Because it means ‘God’s
precious treasure.’”
God kills, I thought then. The same God who
brought us Christmas and the Easter Bunny—he
murdered my mother.
(30) Shortly after that Sunday, I brought up
my Korean mother again, asking about the car
accident, how it happened, exactly—was it like
Phil Haag’s father, who fell asleep at the wheel? Or
like our plumber’s teenage son who drove into a
(35) semi head-on?
“Sarah,” Christine said patiently, looking
up from the chopping board, where she was
slicing carrot discs for pot roast. “We really knew
nothing about her. I’m your mommy. Let’s not 
(40) talk about this any more, it makes me sad.” She
made little crying motions, pretending to wipe
away tears, the same thing she did when I was
bad, to show how I had disappointed her.
I grew up in a house in which Korea had
(45) always been the oddly charged word, never to be
mentioned in connection to me, the same way
we never said “Uncle Henry” and “alcoholic” in
the same sentence. It was almost as if Ken and
Christine thought I needed to be protected from
(50) it, the way small children need to be protected
from boors itching to tell them that Santa Claus
is not real.The ban on Korea extended even to
the aforementioned Uncle Henry, who was then
deprived of his war stories at our Memorial Day
(55) cookouts. Although he proudly wore his felt VFW
hat with its flurry of pins, including ones from his
tour “overseas,” Christine or Ken would quietly
slip him some of his favorite Pabst or Schlitz, and
in return he’d set up residence in the lawn chair
(60) at the far corner of our yard, away from everyone.
Somewhere back in the fuzzy clot of my
teens (now, I’m at the worldly-wise age of almost-
twenty), the ’88 Summer Olympics were held
in Seoul. We couldn’t buck the Thorson family
(65) tradition of watching absolutely everything (that
winter we’d raptly watched curling, for God’s
sakes!). But I was aware that pains were taken
to modulate voices, vocal cords twisted to an
excruciating, studied casualness until Korea
(70) came out “Korea,” exactly the same way we’d say
“Russia” or “Carl Lewis” or “Flo-Jo.”
Then Bryant Gumbel invaded our living room
with his special segment on how Korea, one of the
four “Little Tiger” economic miracle countries,
(75) was so enterprising that it had even made an
export product out of its babies. Since the Korean
War, more than a hundred thousand children,
Made-in-Korea stamped on their foreheads, had
left the country, their adoption fees fattening the
(80) government coffers.
Top that, Singapore! Gumbel’s cheery smirk
seemed to say.
“Well, Sarah’s really American, not Kor—”
Amanda began, until the look on Christine’s
(85) face—despairing, fierce—stopped her.
We invent what becomes us.

Q. The passage suggests that Uncle Henry’s role in the Thorson family is that of

Detailed Solution for OneTime: Digital SAT Mock Test - 6 - Question 21

In lines 44–60, the narrator describes her Uncle Henry as an “alcoholic” (line 47) who sat drinking at family cookouts at the far corner of our yard, away from everyone (line 60). This is treatment appropriate to a pitiable embarrassment rather than a stern patriarch or noble hero. There is also no indication, despite Sarah’s parents’ discomfort with discussing her heritage, that Uncle Henry is a bigoted lout.

OneTime: Digital SAT Mock Test - 6 - Question 22

Question based on the following passage.
This passage is adapted from Marie Myung-Ok Lee, Somebody’s Daughter. ©2006 Beacon Press. The story is about a Korean-American girl adopted by an American family and raised in the Midwest.

When I was eight, they told me that my mother’s
death was preordained. She had been murdered.
One Sunday after service, our minister,
Reverend Jansen of the Lutheran Church of the
(5) Good Shepherd, bent down in a cloud of Aqua
Velva to explain. We had been learning in Sunday
school about Heaven and Hell, and in the middle
of class I had fallen into a panic, wondering how
I would recognize my Korean mother when I saw
(10) her in Heaven—or in Hell, if perhaps she and I
both sinned too much.
Not to worry, I was told.
“God called your Korean parents home so
that you could become the daughter of your
(15) mother and father,” he said, his eyes sliding
sidewise, for just a second. His breath smelled
vaguely of toast.
“It was all part of His plan—you see how much
your mommy and daddy love you? When the time
(20) comes, if you’re a very good girl, you, your mommy,
daddy, and your sister, Amanda—the whole
Thorson family—will be in heaven together, thanks
to the Lord’s wonderful and mysterious ways.”
“That’s why we named you Sarah,” Christine
(25) and Ken added. “Because it means ‘God’s
precious treasure.’”
God kills, I thought then. The same God who
brought us Christmas and the Easter Bunny—he
murdered my mother.
(30) Shortly after that Sunday, I brought up
my Korean mother again, asking about the car
accident, how it happened, exactly—was it like
Phil Haag’s father, who fell asleep at the wheel? Or
like our plumber’s teenage son who drove into a
(35) semi head-on?
“Sarah,” Christine said patiently, looking
up from the chopping board, where she was
slicing carrot discs for pot roast. “We really knew
nothing about her. I’m your mommy. Let’s not 
(40) talk about this any more, it makes me sad.” She
made little crying motions, pretending to wipe
away tears, the same thing she did when I was
bad, to show how I had disappointed her.
I grew up in a house in which Korea had
(45) always been the oddly charged word, never to be
mentioned in connection to me, the same way
we never said “Uncle Henry” and “alcoholic” in
the same sentence. It was almost as if Ken and
Christine thought I needed to be protected from
(50) it, the way small children need to be protected
from boors itching to tell them that Santa Claus
is not real.The ban on Korea extended even to
the aforementioned Uncle Henry, who was then
deprived of his war stories at our Memorial Day
(55) cookouts. Although he proudly wore his felt VFW
hat with its flurry of pins, including ones from his
tour “overseas,” Christine or Ken would quietly
slip him some of his favorite Pabst or Schlitz, and
in return he’d set up residence in the lawn chair
(60) at the far corner of our yard, away from everyone.
Somewhere back in the fuzzy clot of my
teens (now, I’m at the worldly-wise age of almost-
twenty), the ’88 Summer Olympics were held
in Seoul. We couldn’t buck the Thorson family
(65) tradition of watching absolutely everything (that
winter we’d raptly watched curling, for God’s
sakes!). But I was aware that pains were taken
to modulate voices, vocal cords twisted to an
excruciating, studied casualness until Korea
(70) came out “Korea,” exactly the same way we’d say
“Russia” or “Carl Lewis” or “Flo-Jo.”
Then Bryant Gumbel invaded our living room
with his special segment on how Korea, one of the
four “Little Tiger” economic miracle countries,
(75) was so enterprising that it had even made an
export product out of its babies. Since the Korean
War, more than a hundred thousand children,
Made-in-Korea stamped on their foreheads, had
left the country, their adoption fees fattening the
(80) government coffers.
Top that, Singapore! Gumbel’s cheery smirk
seemed to say.
“Well, Sarah’s really American, not Kor—”
Amanda began, until the look on Christine’s
(85) face—despairing, fierce—stopped her.
We invent what becomes us.

Q. The “cheery smirk” (line 81) is taken by the narrator to indicate Gumbel’s

Detailed Solution for OneTime: Digital SAT Mock Test - 6 - Question 22

The reference to Bryant Gumbel’s cheery smirk (line 81) follows the description of his television segment during the Olympic games about how Korea had become one of the economic miracle countries (line 74). According to the narrator, the cheery smirk seemed to say Top that, Singapore! thereby indicating that he admired Korea’s ability to compete economically with other strong countries.

OneTime: Digital SAT Mock Test - 6 - Question 23

Question based on the following passage and supplementary material.
This passage is adapted from G. M. Fitzhenry, “Baby Pictures of the Universe.” ©2015 by College Hill Coaching.

At the breathtaking Gettysburg Cyclorama,
a 377-foot-long, 42-foot-high painting of the bloody
1863 Battle of Gettysburg, visitors can turn in
every direction and feel as if they have been thrust
(5) into the midst of perhaps the most important
battle in American history, a snapshot of a chaotic
chapter in the early life of a nation. Yet right now
you sit in the midst of an even more spectacular
cyclorama of an even more cataclysmic historical
(10) event that took place billions of years ago.
Unfortunately, to appreciate its full splendor, you
would have to be able to see microwaves, which
are invisible to our human eyes.
This real-life cyclorama is the cosmic
(15) microwave background (CMB) radiation, a
13-billion-year-old panoramic snapshot of
the universe as it appeared the moment it first
released its primordial photons. Although it is an
astonishingly detailed confirmation of the Big Bang
(20) theory, it is not actually a picture of the Big Bang.
On a human scale, it corresponds not to the instant
of childbirth, but rather the moment a swaddled
one-day-old opens its eyes and keeps them open.
For the first 380,000 years of its life (a mere
(25) blink of an eye in cosmic history), the universe was
“invisible” because its photons—the particles that
are emitted from an object or event and that must
reach a detector in order for us to “see” it—were
trapped in a hot, opaque fog of hydrogen plasma.
(30) Only when this super-heated plasma cooled to the
point where protons and electrons could combine
to form hydrogen atoms—a period called the
“epoch of recombination”—did these photons
begin to travel unimpeded through the universe.
(35) Some of those photons, having traveled for half a
billion generations, are just now reaching us.
One of the most striking aspects of the CMB
radiation is its near-uniformity, or “isotropism.” No
matter where we look in the sky, the temperature of
(40) the CMB radiation varies by no more than one part in
100,000. It’s almost impossible to find another
real-life example of such thermal homogeneity.
This uniformity is somewhat counterintuitive:
the remnants of most explosions seem to spread
(45) out in a spherical but non-uniform “debris field.”
For instance, the embers of a firework explosion
are confined to a region around the explosion, but
nowhere else. So why is the CMB radiation still
found everywhere in the universe, and not just on
(50) its “edges?” The first reason is that the universe
has no edges: it is “boundless,” just as the surface
of a sphere is boundless. The second reason is that
the CMB radiation did not originate from just one
point in space, but from virtually every point in
(55) space. Thus, every point in the modern universe is
not only equally likely to be the source of the CMB
radiation, it is also equally likely to be the current
location of the CMB radiation.
This uniformity was predicted in a theory
(60) published by George Gamow in 1948. His theory
also made two other predictions that have
been confirmed to astonishing precision by
our current data. First, Gamow predicted that
the CMB radiation should have a distinctive
(65) spectrum known as a “blackbody” curve. Second,
he predicted that the expanding universe would
have cooled this radiation to below 5 degrees
Kelvin today.
(70) The CMB radiation went undetected until
1964, when Arno Penzias and Robert Wilson
at Bell Laboratories in Murray Hill, New Jersey
became troubled by persistent background noise
in a radio telescope that they had just built. Their
initial explanation was that it was due to a “white
(75) dielectric substance,” more commonly known as
pigeon droppings. Remarkably, less than 40 miles
away, Princeton researchers Robert Dicke and
Dave Wilkinson had been searching for evidence
supporting Gamow’s predictions, and instantly
(80) knew of a much better explanation for the noise.
Penzias and Wilson shared the 1978 Nobel Prize in
physics for their discovery of the CMB radiation.
Since then, much more careful observations,
made by the NASA Cosmic Background Explorer
(85) (COBE) and the Wilkinson Microwave Anisotropy
Probe (WMAP) have confirmed that the CMB
radiation indeed has a nearly perfect blackbody
spectrum corresponding to a temperature of
2.725° Kelvin, barely more than 2 degrees from
(90) Gamow’s guess. In addition to confirming many
aspects of the Big Bang theory, these data have
also helped scientists calibrate the age of the
universe (13.772 ± 0.059 billion years), gauge
the speed at which the universe is expanding,
(95) and even verify the existence of “dark energy,”
the mysterious energy that propelled the rapid
expansion of the early universe.


Q. This passage is primarily concerned with

Detailed Solution for OneTime: Digital SAT Mock Test - 6 - Question 23

The passage as a whole describes the spectacular cyclorama (line 9) that is known as the cosmic microwave background (CMB) radiation, a 13 billion year-old panoramic snapshot of the universe as it appeared the moment it first released its primordial photons (lines 14–18). It then goes on to discuss the precise measurements that scientists have taken of this radiation and what they tell us about the early universe. In other words, the passage as a whole is discussing the analysis and significance of a cosmological phenomenon.

OneTime: Digital SAT Mock Test - 6 - Question 24

Question based on the following passage and supplementary material.
This passage is adapted from G. M. Fitzhenry, “Baby Pictures of the Universe.” ©2015 by College Hill Coaching.

At the breathtaking Gettysburg Cyclorama,
a 377-foot-long, 42-foot-high painting of the bloody
1863 Battle of Gettysburg, visitors can turn in
every direction and feel as if they have been thrust
(5) into the midst of perhaps the most important
battle in American history, a snapshot of a chaotic
chapter in the early life of a nation. Yet right now
you sit in the midst of an even more spectacular
cyclorama of an even more cataclysmic historical
(10) event that took place billions of years ago.
Unfortunately, to appreciate its full splendor, you
would have to be able to see microwaves, which
are invisible to our human eyes.
This real-life cyclorama is the cosmic
(15) microwave background (CMB) radiation, a
13-billion-year-old panoramic snapshot of
the universe as it appeared the moment it first
released its primordial photons. Although it is an
astonishingly detailed confirmation of the Big Bang
(20) theory, it is not actually a picture of the Big Bang.
On a human scale, it corresponds not to the instant
of childbirth, but rather the moment a swaddled
one-day-old opens its eyes and keeps them open.
For the first 380,000 years of its life (a mere
(25) blink of an eye in cosmic history), the universe was
“invisible” because its photons—the particles that
are emitted from an object or event and that must
reach a detector in order for us to “see” it—were
trapped in a hot, opaque fog of hydrogen plasma.
(30) Only when this super-heated plasma cooled to the
point where protons and electrons could combine
to form hydrogen atoms—a period called the
“epoch of recombination”—did these photons
begin to travel unimpeded through the universe.
(35) Some of those photons, having traveled for half a
billion generations, are just now reaching us.
One of the most striking aspects of the CMB
radiation is its near-uniformity, or “isotropism.” No
matter where we look in the sky, the temperature of
(40) the CMB radiation varies by no more than one part in
100,000. It’s almost impossible to find another
real-life example of such thermal homogeneity.
This uniformity is somewhat counterintuitive:
the remnants of most explosions seem to spread
(45) out in a spherical but non-uniform “debris field.”
For instance, the embers of a firework explosion
are confined to a region around the explosion, but
nowhere else. So why is the CMB radiation still
found everywhere in the universe, and not just on
(50) its “edges?” The first reason is that the universe
has no edges: it is “boundless,” just as the surface
of a sphere is boundless. The second reason is that
the CMB radiation did not originate from just one
point in space, but from virtually every point in
(55) space. Thus, every point in the modern universe is
not only equally likely to be the source of the CMB
radiation, it is also equally likely to be the current
location of the CMB radiation.
This uniformity was predicted in a theory
(60) published by George Gamow in 1948. His theory
also made two other predictions that have
been confirmed to astonishing precision by
our current data. First, Gamow predicted that
the CMB radiation should have a distinctive
(65) spectrum known as a “blackbody” curve. Second,
he predicted that the expanding universe would
have cooled this radiation to below 5 degrees
Kelvin today.
(70) The CMB radiation went undetected until
1964, when Arno Penzias and Robert Wilson
at Bell Laboratories in Murray Hill, New Jersey
became troubled by persistent background noise
in a radio telescope that they had just built. Their
initial explanation was that it was due to a “white
(75) dielectric substance,” more commonly known as
pigeon droppings. Remarkably, less than 40 miles
away, Princeton researchers Robert Dicke and
Dave Wilkinson had been searching for evidence
supporting Gamow’s predictions, and instantly
(80) knew of a much better explanation for the noise.
Penzias and Wilson shared the 1978 Nobel Prize in
physics for their discovery of the CMB radiation.
Since then, much more careful observations,
made by the NASA Cosmic Background Explorer
(85) (COBE) and the Wilkinson Microwave Anisotropy
Probe (WMAP) have confirmed that the CMB
radiation indeed has a nearly perfect blackbody
spectrum corresponding to a temperature of
2.725° Kelvin, barely more than 2 degrees from
(90) Gamow’s guess. In addition to confirming many
aspects of the Big Bang theory, these data have
also helped scientists calibrate the age of the
universe (13.772 ± 0.059 billion years), gauge
the speed at which the universe is expanding,
(95) and even verify the existence of “dark energy,”
the mysterious energy that propelled the rapid
expansion of the early universe.


Q. In the context of the passage as a whole, the Gettysburg Cyclorama represents

Detailed Solution for OneTime: Digital SAT Mock Test - 6 - Question 24

The description of the Gettysburg Cyclorama in the first paragraph is used to draw an analogy between two cataclysmic historical event[s] (lines 9–10), one of which we can see with our own eyes and one of which we can only detect with special tools. The answer is not (B), because although this Cyclorama depicts a historic battle, it is not itself a historical precedent (an event that serves as a model for future similar events). Choice (C) is incorrect because the painting is depicted neither as quaint nor anachronistic (out of historical order). Choice (D) is incorrect because although the passage later indicates that the discovery of the CMB was somewhat accidental, the Cyclorama was not.

OneTime: Digital SAT Mock Test - 6 - Question 25

Question based on the following passage and supplementary material.
This passage is adapted from G. M. Fitzhenry, “Baby Pictures of the Universe.” ©2015 by College Hill Coaching.

At the breathtaking Gettysburg Cyclorama,
a 377-foot-long, 42-foot-high painting of the bloody
1863 Battle of Gettysburg, visitors can turn in
every direction and feel as if they have been thrust
(5) into the midst of perhaps the most important
battle in American history, a snapshot of a chaotic
chapter in the early life of a nation. Yet right now
you sit in the midst of an even more spectacular
cyclorama of an even more cataclysmic historical
(10) event that took place billions of years ago.
Unfortunately, to appreciate its full splendor, you
would have to be able to see microwaves, which
are invisible to our human eyes.
This real-life cyclorama is the cosmic
(15) microwave background (CMB) radiation, a
13-billion-year-old panoramic snapshot of
the universe as it appeared the moment it first
released its primordial photons. Although it is an
astonishingly detailed confirmation of the Big Bang
(20) theory, it is not actually a picture of the Big Bang.
On a human scale, it corresponds not to the instant
of childbirth, but rather the moment a swaddled
one-day-old opens its eyes and keeps them open.
For the first 380,000 years of its life (a mere
(25) blink of an eye in cosmic history), the universe was
“invisible” because its photons—the particles that
are emitted from an object or event and that must
reach a detector in order for us to “see” it—were
trapped in a hot, opaque fog of hydrogen plasma.
(30) Only when this super-heated plasma cooled to the
point where protons and electrons could combine
to form hydrogen atoms—a period called the
“epoch of recombination”—did these photons
begin to travel unimpeded through the universe.
(35) Some of those photons, having traveled for half a
billion generations, are just now reaching us.
One of the most striking aspects of the CMB
radiation is its near-uniformity, or “isotropism.” No
matter where we look in the sky, the temperature of
(40) the CMB radiation varies by no more than one part in
100,000. It’s almost impossible to find another
real-life example of such thermal homogeneity.
This uniformity is somewhat counterintuitive:
the remnants of most explosions seem to spread
(45) out in a spherical but non-uniform “debris field.”
For instance, the embers of a firework explosion
are confined to a region around the explosion, but
nowhere else. So why is the CMB radiation still
found everywhere in the universe, and not just on
(50) its “edges?” The first reason is that the universe
has no edges: it is “boundless,” just as the surface
of a sphere is boundless. The second reason is that
the CMB radiation did not originate from just one
point in space, but from virtually every point in
(55) space. Thus, every point in the modern universe is
not only equally likely to be the source of the CMB
radiation, it is also equally likely to be the current
location of the CMB radiation.
This uniformity was predicted in a theory
(60) published by George Gamow in 1948. His theory
also made two other predictions that have
been confirmed to astonishing precision by
our current data. First, Gamow predicted that
the CMB radiation should have a distinctive
(65) spectrum known as a “blackbody” curve. Second,
he predicted that the expanding universe would
have cooled this radiation to below 5 degrees
Kelvin today.
(70) The CMB radiation went undetected until
1964, when Arno Penzias and Robert Wilson
at Bell Laboratories in Murray Hill, New Jersey
became troubled by persistent background noise
in a radio telescope that they had just built. Their
initial explanation was that it was due to a “white
(75) dielectric substance,” more commonly known as
pigeon droppings. Remarkably, less than 40 miles
away, Princeton researchers Robert Dicke and
Dave Wilkinson had been searching for evidence
supporting Gamow’s predictions, and instantly
(80) knew of a much better explanation for the noise.
Penzias and Wilson shared the 1978 Nobel Prize in
physics for their discovery of the CMB radiation.
Since then, much more careful observations,
made by the NASA Cosmic Background Explorer
(85) (COBE) and the Wilkinson Microwave Anisotropy
Probe (WMAP) have confirmed that the CMB
radiation indeed has a nearly perfect blackbody
spectrum corresponding to a temperature of
2.725° Kelvin, barely more than 2 degrees from
(90) Gamow’s guess. In addition to confirming many
aspects of the Big Bang theory, these data have
also helped scientists calibrate the age of the
universe (13.772 ± 0.059 billion years), gauge
the speed at which the universe is expanding,
(95) and even verify the existence of “dark energy,”
the mysterious energy that propelled the rapid
expansion of the early universe.


Q. Lines 11–13 (“Unfortunately . . . human eyes”) convey the author’s disappointment in

Detailed Solution for OneTime: Digital SAT Mock Test - 6 - Question 25

The author indicates that to appreciate [the] full splendor[of the Cosmic Background Radiation], you would have to be able to see microwaves (lines 11–13). In other words, the disappointment is in the fact that we can’t see the spectacular cyclorama (line 8) that is the cosmic microwave background; it is an inaccessible phenomenon, at least to our naked eyes.

OneTime: Digital SAT Mock Test - 6 - Question 26

Question based on the following passage and supplementary material.
This passage is adapted from G. M. Fitzhenry, “Baby Pictures of the Universe.” ©2015 by College Hill Coaching.

At the breathtaking Gettysburg Cyclorama,
a 377-foot-long, 42-foot-high painting of the bloody
1863 Battle of Gettysburg, visitors can turn in
every direction and feel as if they have been thrust
(5) into the midst of perhaps the most important
battle in American history, a snapshot of a chaotic
chapter in the early life of a nation. Yet right now
you sit in the midst of an even more spectacular
cyclorama of an even more cataclysmic historical
(10) event that took place billions of years ago.
Unfortunately, to appreciate its full splendor, you
would have to be able to see microwaves, which
are invisible to our human eyes.
This real-life cyclorama is the cosmic
(15) microwave background (CMB) radiation, a
13-billion-year-old panoramic snapshot of
the universe as it appeared the moment it first
released its primordial photons. Although it is an
astonishingly detailed confirmation of the Big Bang
(20) theory, it is not actually a picture of the Big Bang.
On a human scale, it corresponds not to the instant
of childbirth, but rather the moment a swaddled
one-day-old opens its eyes and keeps them open.
For the first 380,000 years of its life (a mere
(25) blink of an eye in cosmic history), the universe was
“invisible” because its photons—the particles that
are emitted from an object or event and that must
reach a detector in order for us to “see” it—were
trapped in a hot, opaque fog of hydrogen plasma.
(30) Only when this super-heated plasma cooled to the
point where protons and electrons could combine
to form hydrogen atoms—a period called the
“epoch of recombination”—did these photons
begin to travel unimpeded through the universe.
(35) Some of those photons, having traveled for half a
billion generations, are just now reaching us.
One of the most striking aspects of the CMB
radiation is its near-uniformity, or “isotropism.” No
matter where we look in the sky, the temperature of
(40) the CMB radiation varies by no more than one part in
100,000. It’s almost impossible to find another
real-life example of such thermal homogeneity.
This uniformity is somewhat counterintuitive:
the remnants of most explosions seem to spread
(45) out in a spherical but non-uniform “debris field.”
For instance, the embers of a firework explosion
are confined to a region around the explosion, but
nowhere else. So why is the CMB radiation still
found everywhere in the universe, and not just on
(50) its “edges?” The first reason is that the universe
has no edges: it is “boundless,” just as the surface
of a sphere is boundless. The second reason is that
the CMB radiation did not originate from just one
point in space, but from virtually every point in
(55) space. Thus, every point in the modern universe is
not only equally likely to be the source of the CMB
radiation, it is also equally likely to be the current
location of the CMB radiation.
This uniformity was predicted in a theory
(60) published by George Gamow in 1948. His theory
also made two other predictions that have
been confirmed to astonishing precision by
our current data. First, Gamow predicted that
the CMB radiation should have a distinctive
(65) spectrum known as a “blackbody” curve. Second,
he predicted that the expanding universe would
have cooled this radiation to below 5 degrees
Kelvin today.
(70) The CMB radiation went undetected until
1964, when Arno Penzias and Robert Wilson
at Bell Laboratories in Murray Hill, New Jersey
became troubled by persistent background noise
in a radio telescope that they had just built. Their
initial explanation was that it was due to a “white
(75) dielectric substance,” more commonly known as
pigeon droppings. Remarkably, less than 40 miles
away, Princeton researchers Robert Dicke and
Dave Wilkinson had been searching for evidence
supporting Gamow’s predictions, and instantly
(80) knew of a much better explanation for the noise.
Penzias and Wilson shared the 1978 Nobel Prize in
physics for their discovery of the CMB radiation.
Since then, much more careful observations,
made by the NASA Cosmic Background Explorer
(85) (COBE) and the Wilkinson Microwave Anisotropy
Probe (WMAP) have confirmed that the CMB
radiation indeed has a nearly perfect blackbody
spectrum corresponding to a temperature of
2.725° Kelvin, barely more than 2 degrees from
(90) Gamow’s guess. In addition to confirming many
aspects of the Big Bang theory, these data have
also helped scientists calibrate the age of the
universe (13.772 ± 0.059 billion years), gauge
the speed at which the universe is expanding,
(95) and even verify the existence of “dark energy,”
the mysterious energy that propelled the rapid
expansion of the early universe.


Q. The quotation marks around the words “invisible” (line 26) and “see” (line 28) serve primarily to

Detailed Solution for OneTime: Digital SAT Mock Test - 6 - Question 26

The discussion in lines 24–28 concerns the emergence of the first photons (light particles) in the early universe. In saying that the universe was “invisible” (lines 25–26), the author means that photons—the particles that are required for us to be able to detect something visually— did not yet exist. Calling the early universe “invisible” is somewhat inappropriate, since there were no eyes to see it anyway during that stage in its development, so the quotes are drawing attention to the fact that these terms are being used to make a technical point a bit clearer by using common words that correspond with our everyday experience.

OneTime: Digital SAT Mock Test - 6 - Question 27

Question based on the following passage and supplementary material.
This passage is adapted from G. M. Fitzhenry, “Baby Pictures of the Universe.” ©2015 by College Hill Coaching.

At the breathtaking Gettysburg Cyclorama,
a 377-foot-long, 42-foot-high painting of the bloody
1863 Battle of Gettysburg, visitors can turn in
every direction and feel as if they have been thrust
(5) into the midst of perhaps the most important
battle in American history, a snapshot of a chaotic
chapter in the early life of a nation. Yet right now
you sit in the midst of an even more spectacular
cyclorama of an even more cataclysmic historical
(10) event that took place billions of years ago.
Unfortunately, to appreciate its full splendor, you
would have to be able to see microwaves, which
are invisible to our human eyes.
This real-life cyclorama is the cosmic
(15) microwave background (CMB) radiation, a
13-billion-year-old panoramic snapshot of
the universe as it appeared the moment it first
released its primordial photons. Although it is an
astonishingly detailed confirmation of the Big Bang
(20) theory, it is not actually a picture of the Big Bang.
On a human scale, it corresponds not to the instant
of childbirth, but rather the moment a swaddled
one-day-old opens its eyes and keeps them open.
For the first 380,000 years of its life (a mere
(25) blink of an eye in cosmic history), the universe was
“invisible” because its photons—the particles that
are emitted from an object or event and that must
reach a detector in order for us to “see” it—were
trapped in a hot, opaque fog of hydrogen plasma.
(30) Only when this super-heated plasma cooled to the
point where protons and electrons could combine
to form hydrogen atoms—a period called the
“epoch of recombination”—did these photons
begin to travel unimpeded through the universe.
(35) Some of those photons, having traveled for half a
billion generations, are just now reaching us.
One of the most striking aspects of the CMB
radiation is its near-uniformity, or “isotropism.” No
matter where we look in the sky, the temperature of
(40) the CMB radiation varies by no more than one part in
100,000. It’s almost impossible to find another
real-life example of such thermal homogeneity.
This uniformity is somewhat counterintuitive:
the remnants of most explosions seem to spread
(45) out in a spherical but non-uniform “debris field.”
For instance, the embers of a firework explosion
are confined to a region around the explosion, but
nowhere else. So why is the CMB radiation still
found everywhere in the universe, and not just on
(50) its “edges?” The first reason is that the universe
has no edges: it is “boundless,” just as the surface
of a sphere is boundless. The second reason is that
the CMB radiation did not originate from just one
point in space, but from virtually every point in
(55) space. Thus, every point in the modern universe is
not only equally likely to be the source of the CMB
radiation, it is also equally likely to be the current
location of the CMB radiation.
This uniformity was predicted in a theory
(60) published by George Gamow in 1948. His theory
also made two other predictions that have
been confirmed to astonishing precision by
our current data. First, Gamow predicted that
the CMB radiation should have a distinctive
(65) spectrum known as a “blackbody” curve. Second,
he predicted that the expanding universe would
have cooled this radiation to below 5 degrees
Kelvin today.
(70) The CMB radiation went undetected until
1964, when Arno Penzias and Robert Wilson
at Bell Laboratories in Murray Hill, New Jersey
became troubled by persistent background noise
in a radio telescope that they had just built. Their
initial explanation was that it was due to a “white
(75) dielectric substance,” more commonly known as
pigeon droppings. Remarkably, less than 40 miles
away, Princeton researchers Robert Dicke and
Dave Wilkinson had been searching for evidence
supporting Gamow’s predictions, and instantly
(80) knew of a much better explanation for the noise.
Penzias and Wilson shared the 1978 Nobel Prize in
physics for their discovery of the CMB radiation.
Since then, much more careful observations,
made by the NASA Cosmic Background Explorer
(85) (COBE) and the Wilkinson Microwave Anisotropy
Probe (WMAP) have confirmed that the CMB
radiation indeed has a nearly perfect blackbody
spectrum corresponding to a temperature of
2.725° Kelvin, barely more than 2 degrees from
(90) Gamow’s guess. In addition to confirming many
aspects of the Big Bang theory, these data have
also helped scientists calibrate the age of the
universe (13.772 ± 0.059 billion years), gauge
the speed at which the universe is expanding,
(95) and even verify the existence of “dark energy,”
the mysterious energy that propelled the rapid
expansion of the early universe.


Q. The “moment a swaddled one-day-old opens its eyes” (lines 22–23) corresponds to the instant that

Detailed Solution for OneTime: Digital SAT Mock Test - 6 - Question 27

The moment a swaddled one-day-old opens its eyes (lines 22–23) refers to the moment [the early universe] first released its primordial photons (lines 17–18) which we now refer to as the cosmic microwave background radiation. The discussion in the next paragraph (lines 24–36) explains that these early photons were previously trapped in an opaque fog of hydrogen plasma (line 29). Choices (A) and (D) are incorrect because this moment describes when the photons were released, not when they were first discovered by humans. Choice (B) is incorrect because these particles, as it is explained in the third paragraph, were released 380,000 years after the Big Bang.

OneTime: Digital SAT Mock Test - 6 - Question 28

Question based on the following passage and supplementary material.
This passage is adapted from G. M. Fitzhenry, “Baby Pictures of the Universe.” ©2015 by College Hill Coaching.

At the breathtaking Gettysburg Cyclorama,
a 377-foot-long, 42-foot-high painting of the bloody
1863 Battle of Gettysburg, visitors can turn in
every direction and feel as if they have been thrust
(5) into the midst of perhaps the most important
battle in American history, a snapshot of a chaotic
chapter in the early life of a nation. Yet right now
you sit in the midst of an even more spectacular
cyclorama of an even more cataclysmic historical
(10) event that took place billions of years ago.
Unfortunately, to appreciate its full splendor, you
would have to be able to see microwaves, which
are invisible to our human eyes.
This real-life cyclorama is the cosmic
(15) microwave background (CMB) radiation, a
13-billion-year-old panoramic snapshot of
the universe as it appeared the moment it first
released its primordial photons. Although it is an
astonishingly detailed confirmation of the Big Bang
(20) theory, it is not actually a picture of the Big Bang.
On a human scale, it corresponds not to the instant
of childbirth, but rather the moment a swaddled
one-day-old opens its eyes and keeps them open.
For the first 380,000 years of its life (a mere
(25) blink of an eye in cosmic history), the universe was
“invisible” because its photons—the particles that
are emitted from an object or event and that must
reach a detector in order for us to “see” it—were
trapped in a hot, opaque fog of hydrogen plasma.
(30) Only when this super-heated plasma cooled to the
point where protons and electrons could combine
to form hydrogen atoms—a period called the
“epoch of recombination”—did these photons
begin to travel unimpeded through the universe.
(35) Some of those photons, having traveled for half a
billion generations, are just now reaching us.
One of the most striking aspects of the CMB
radiation is its near-uniformity, or “isotropism.” No
matter where we look in the sky, the temperature of
(40) the CMB radiation varies by no more than one part in
100,000. It’s almost impossible to find another
real-life example of such thermal homogeneity.
This uniformity is somewhat counterintuitive:
the remnants of most explosions seem to spread
(45) out in a spherical but non-uniform “debris field.”
For instance, the embers of a firework explosion
are confined to a region around the explosion, but
nowhere else. So why is the CMB radiation still
found everywhere in the universe, and not just on
(50) its “edges?” The first reason is that the universe
has no edges: it is “boundless,” just as the surface
of a sphere is boundless. The second reason is that
the CMB radiation did not originate from just one
point in space, but from virtually every point in
(55) space. Thus, every point in the modern universe is
not only equally likely to be the source of the CMB
radiation, it is also equally likely to be the current
location of the CMB radiation.
This uniformity was predicted in a theory
(60) published by George Gamow in 1948. His theory
also made two other predictions that have
been confirmed to astonishing precision by
our current data. First, Gamow predicted that
the CMB radiation should have a distinctive
(65) spectrum known as a “blackbody” curve. Second,
he predicted that the expanding universe would
have cooled this radiation to below 5 degrees
Kelvin today.
(70) The CMB radiation went undetected until
1964, when Arno Penzias and Robert Wilson
at Bell Laboratories in Murray Hill, New Jersey
became troubled by persistent background noise
in a radio telescope that they had just built. Their
initial explanation was that it was due to a “white
(75) dielectric substance,” more commonly known as
pigeon droppings. Remarkably, less than 40 miles
away, Princeton researchers Robert Dicke and
Dave Wilkinson had been searching for evidence
supporting Gamow’s predictions, and instantly
(80) knew of a much better explanation for the noise.
Penzias and Wilson shared the 1978 Nobel Prize in
physics for their discovery of the CMB radiation.
Since then, much more careful observations,
made by the NASA Cosmic Background Explorer
(85) (COBE) and the Wilkinson Microwave Anisotropy
Probe (WMAP) have confirmed that the CMB
radiation indeed has a nearly perfect blackbody
spectrum corresponding to a temperature of
2.725° Kelvin, barely more than 2 degrees from
(90) Gamow’s guess. In addition to confirming many
aspects of the Big Bang theory, these data have
also helped scientists calibrate the age of the
universe (13.772 ± 0.059 billion years), gauge
the speed at which the universe is expanding,
(95) and even verify the existence of “dark energy,”
the mysterious energy that propelled the rapid
expansion of the early universe.


Q. In line 64, “distinctive” most nearly means

Detailed Solution for OneTime: Digital SAT Mock Test - 6 - Question 28

The distinctive spectrum (lines 64–65) refers to the precise “blackbody” curve for 2.75° Kelvin as shown in Figure 1. It is the particular set of wavelength intensities that distinguish blackbody radiation from ordinary radiation, and confirm Gamow’s theory about the origin of the signals detected at Murray Hill.

OneTime: Digital SAT Mock Test - 6 - Question 29

Question based on the following passage and supplementary material.
This passage is adapted from G. M. Fitzhenry, “Baby Pictures of the Universe.” ©2015 by College Hill Coaching.

At the breathtaking Gettysburg Cyclorama,
a 377-foot-long, 42-foot-high painting of the bloody
1863 Battle of Gettysburg, visitors can turn in
every direction and feel as if they have been thrust
(5) into the midst of perhaps the most important
battle in American history, a snapshot of a chaotic
chapter in the early life of a nation. Yet right now
you sit in the midst of an even more spectacular
cyclorama of an even more cataclysmic historical
(10) event that took place billions of years ago.
Unfortunately, to appreciate its full splendor, you
would have to be able to see microwaves, which
are invisible to our human eyes.
This real-life cyclorama is the cosmic
(15) microwave background (CMB) radiation, a
13-billion-year-old panoramic snapshot of
the universe as it appeared the moment it first
released its primordial photons. Although it is an
astonishingly detailed confirmation of the Big Bang
(20) theory, it is not actually a picture of the Big Bang.
On a human scale, it corresponds not to the instant
of childbirth, but rather the moment a swaddled
one-day-old opens its eyes and keeps them open.
For the first 380,000 years of its life (a mere
(25) blink of an eye in cosmic history), the universe was
“invisible” because its photons—the particles that
are emitted from an object or event and that must
reach a detector in order for us to “see” it—were
trapped in a hot, opaque fog of hydrogen plasma.
(30) Only when this super-heated plasma cooled to the
point where protons and electrons could combine
to form hydrogen atoms—a period called the
“epoch of recombination”—did these photons
begin to travel unimpeded through the universe.
(35) Some of those photons, having traveled for half a
billion generations, are just now reaching us.
One of the most striking aspects of the CMB
radiation is its near-uniformity, or “isotropism.” No
matter where we look in the sky, the temperature of
(40) the CMB radiation varies by no more than one part in
100,000. It’s almost impossible to find another
real-life example of such thermal homogeneity.
This uniformity is somewhat counterintuitive:
the remnants of most explosions seem to spread
(45) out in a spherical but non-uniform “debris field.”
For instance, the embers of a firework explosion
are confined to a region around the explosion, but
nowhere else. So why is the CMB radiation still
found everywhere in the universe, and not just on
(50) its “edges?” The first reason is that the universe
has no edges: it is “boundless,” just as the surface
of a sphere is boundless. The second reason is that
the CMB radiation did not originate from just one
point in space, but from virtually every point in
(55) space. Thus, every point in the modern universe is
not only equally likely to be the source of the CMB
radiation, it is also equally likely to be the current
location of the CMB radiation.
This uniformity was predicted in a theory
(60) published by George Gamow in 1948. His theory
also made two other predictions that have
been confirmed to astonishing precision by
our current data. First, Gamow predicted that
the CMB radiation should have a distinctive
(65) spectrum known as a “blackbody” curve. Second,
he predicted that the expanding universe would
have cooled this radiation to below 5 degrees
Kelvin today.
(70) The CMB radiation went undetected until
1964, when Arno Penzias and Robert Wilson
at Bell Laboratories in Murray Hill, New Jersey
became troubled by persistent background noise
in a radio telescope that they had just built. Their
initial explanation was that it was due to a “white
(75) dielectric substance,” more commonly known as
pigeon droppings. Remarkably, less than 40 miles
away, Princeton researchers Robert Dicke and
Dave Wilkinson had been searching for evidence
supporting Gamow’s predictions, and instantly
(80) knew of a much better explanation for the noise.
Penzias and Wilson shared the 1978 Nobel Prize in
physics for their discovery of the CMB radiation.
Since then, much more careful observations,
made by the NASA Cosmic Background Explorer
(85) (COBE) and the Wilkinson Microwave Anisotropy
Probe (WMAP) have confirmed that the CMB
radiation indeed has a nearly perfect blackbody
spectrum corresponding to a temperature of
2.725° Kelvin, barely more than 2 degrees from
(90) Gamow’s guess. In addition to confirming many
aspects of the Big Bang theory, these data have
also helped scientists calibrate the age of the
universe (13.772 ± 0.059 billion years), gauge
the speed at which the universe is expanding,
(95) and even verify the existence of “dark energy,”
the mysterious energy that propelled the rapid
expansion of the early universe.


Q. Which of the following can be inferred about the work that earned Penzias and Wilson the Nobel Prize?

Detailed Solution for OneTime: Digital SAT Mock Test - 6 - Question 29

The passage states that Penzias and Wilson were initially troubled (line 72) by the signals that turned out to be from the CMB radiation, and in fact mistakenly attributed them to pigeon droppings (line 76). This indicates that they were not looking for these signals, nor did they know how to interpret them. The work they did to receive the Nobel prize, therefore, was the result of an accidental discovery.

OneTime: Digital SAT Mock Test - 6 - Question 30

Question based on the following passage and supplementary material.
This passage is adapted from G. M. Fitzhenry, “Baby Pictures of the Universe.” ©2015 by College Hill Coaching.

At the breathtaking Gettysburg Cyclorama,
a 377-foot-long, 42-foot-high painting of the bloody
1863 Battle of Gettysburg, visitors can turn in
every direction and feel as if they have been thrust
(5) into the midst of perhaps the most important
battle in American history, a snapshot of a chaotic
chapter in the early life of a nation. Yet right now
you sit in the midst of an even more spectacular
cyclorama of an even more cataclysmic historical
(10) event that took place billions of years ago.
Unfortunately, to appreciate its full splendor, you
would have to be able to see microwaves, which
are invisible to our human eyes.
This real-life cyclorama is the cosmic
(15) microwave background (CMB) radiation, a
13-billion-year-old panoramic snapshot of
the universe as it appeared the moment it first
released its primordial photons. Although it is an
astonishingly detailed confirmation of the Big Bang
(20) theory, it is not actually a picture of the Big Bang.
On a human scale, it corresponds not to the instant
of childbirth, but rather the moment a swaddled
one-day-old opens its eyes and keeps them open.
For the first 380,000 years of its life (a mere
(25) blink of an eye in cosmic history), the universe was
“invisible” because its photons—the particles that
are emitted from an object or event and that must
reach a detector in order for us to “see” it—were
trapped in a hot, opaque fog of hydrogen plasma.
(30) Only when this super-heated plasma cooled to the
point where protons and electrons could combine
to form hydrogen atoms—a period called the
“epoch of recombination”—did these photons
begin to travel unimpeded through the universe.
(35) Some of those photons, having traveled for half a
billion generations, are just now reaching us.
One of the most striking aspects of the CMB
radiation is its near-uniformity, or “isotropism.” No
matter where we look in the sky, the temperature of
(40) the CMB radiation varies by no more than one part in
100,000. It’s almost impossible to find another
real-life example of such thermal homogeneity.
This uniformity is somewhat counterintuitive:
the remnants of most explosions seem to spread
(45) out in a spherical but non-uniform “debris field.”
For instance, the embers of a firework explosion
are confined to a region around the explosion, but
nowhere else. So why is the CMB radiation still
found everywhere in the universe, and not just on
(50) its “edges?” The first reason is that the universe
has no edges: it is “boundless,” just as the surface
of a sphere is boundless. The second reason is that
the CMB radiation did not originate from just one
point in space, but from virtually every point in
(55) space. Thus, every point in the modern universe is
not only equally likely to be the source of the CMB
radiation, it is also equally likely to be the current
location of the CMB radiation.
This uniformity was predicted in a theory
(60) published by George Gamow in 1948. His theory
also made two other predictions that have
been confirmed to astonishing precision by
our current data. First, Gamow predicted that
the CMB radiation should have a distinctive
(65) spectrum known as a “blackbody” curve. Second,
he predicted that the expanding universe would
have cooled this radiation to below 5 degrees
Kelvin today.
(70) The CMB radiation went undetected until
1964, when Arno Penzias and Robert Wilson
at Bell Laboratories in Murray Hill, New Jersey
became troubled by persistent background noise
in a radio telescope that they had just built. Their
initial explanation was that it was due to a “white
(75) dielectric substance,” more commonly known as
pigeon droppings. Remarkably, less than 40 miles
away, Princeton researchers Robert Dicke and
Dave Wilkinson had been searching for evidence
supporting Gamow’s predictions, and instantly
(80) knew of a much better explanation for the noise.
Penzias and Wilson shared the 1978 Nobel Prize in
physics for their discovery of the CMB radiation.
Since then, much more careful observations,
made by the NASA Cosmic Background Explorer
(85) (COBE) and the Wilkinson Microwave Anisotropy
Probe (WMAP) have confirmed that the CMB
radiation indeed has a nearly perfect blackbody
spectrum corresponding to a temperature of
2.725° Kelvin, barely more than 2 degrees from
(90) Gamow’s guess. In addition to confirming many
aspects of the Big Bang theory, these data have
also helped scientists calibrate the age of the
universe (13.772 ± 0.059 billion years), gauge
the speed at which the universe is expanding,
(95) and even verify the existence of “dark energy,”
the mysterious energy that propelled the rapid
expansion of the early universe.


Q. Which choice provides the best evidence for the answer to the previous question?

Detailed Solution for OneTime: Digital SAT Mock Test - 6 - Question 30

As the explanation of question 29 indicates, lines 73–76 indicate that Penzias and Wilson did not understand the nature of the signals they were receiving, atttributing them erroneously to pigeon droppings.

View more questions
8 docs|22 tests
Information about OneTime: Digital SAT Mock Test - 6 Page
In this test you can find the Exam questions for OneTime: Digital SAT Mock Test - 6 solved & explained in the simplest way possible. Besides giving Questions and answers for OneTime: Digital SAT Mock Test - 6, EduRev gives you an ample number of Online tests for practice

Top Courses for SAT

Download as PDF

Top Courses for SAT